You are on page 1of 108

Enunciado Auxiliar No1 FI2A2

Prof. Aux.: Felipe L. Benavides


Fecha: Lunes 4 de Agosto de 2008

I. Problemas
Problema 1
Consideremos una cinta infinita de densidad superficial y ancho W . En alg
un punto de la cinta existe un agujero
circular de radio R, con 2R < W . Calcular el campo electrico en el eje de simetra del agujero mencionado.

Figura No 1.
Indicaci
on: Considere conocido el campo electrico producido por un disco de radio conocido y densidad de carga
superficial constante.
Problema 2
Dada una fuente cargada consistente en:
1. Un plano infinito (), cargado con densidad de carga uniforme .
2. Una recta infinita, (L1 ), cargada con densidad uniforme , que forma un angulo agudo 2 con el plano.
Calcular el campo electrico total en un punto P sobre una segunda recta, (L2 ), que bisecta el angulo entre la recta
y el plano. El punto P est
a a altura h sobre el plano.

Figura No 2.

Problema 3
En el interior de una esfera de radio a y centrada en el origen, se tiene una distribucion de carga dada por
(r a) =

5k 2
r , (r > a) = 0
4a5

Obtener:
1. Carga total de la esfera.
2. Campo electrico en todas partes.
3. Potencial electrico en todas partes.
4. Divergencia del campo electrico dentro y fuera de la esfera.
Problema 4
Un cilindro infinito de radio a tiene densidad de carga volumetrica desconocida aunque se sabe que ella tiene simetra
de rotaci
on con respecto al eje del sistema. El cilindro esta rodeado de espacio vaco. Se sabe que el campo electrico
en el interior del cilindro, expresado en coordenadas cilndricas, esta dado por:
A 2

E =

30
donde A es una constante conocida.
Determine:
1. Densidad de carga en todo el espacio.
2. Campo electrico fuera del cilindro.
3. Potencial V en todas partes y tal que se cumpla que V ( = a) = 0.

Resoluci
on Auxiliar No1 FI2A2
Prof. Auxiliar: Felipe L. Benavides
Fecha: Lunes 4 de Agosto, 2008.

I. Problemas
Problema 1
Dada la complejidad geometrica del problema, facilitaremos el calculo con el ppio. de superposicion, obteniendo
el valor deseado como resta entre los valores de campo electrico para la cinta completamente cargada, y un disco,
tambien totalmente cargado. Para este u
ltimo, consideraremos el resultado obtenido en el apunte y visto en c
atedra,
que determina que dicho campo electrico es


z
z

k
E =
20
kzk
z 2 + R2
con R el radio del disco. As, s
olo necesitamos obtener el primer campo electrico y restar.
Por definici
on:
1
40


=
E (
r = z k)

r
r0

dq(
r 0)

k r r 0 k3


Al parametrizar, tendremos que:
r = z k;
r 0 = x0i + y 0 j; dq(
r 0 ) = dx0 dy 0 con w2 x

w
2,

y y

Entonces

E =

1
40

w
2

(z k x0i y 0 j)dx0 dy 0
3

(z 2 + x02 + y 02 ) 2

w
2

De los tres terminos generados, por simetra, solo el primero es no nulo, pues se trata en los dos casos siguientes
de integrales de funciones impares en intervalo simetrico. El calculo se reduce a:
z k

E =
40

w
2

w
2

dx0 dy 0
3

(z 2 + x02 + y 02 ) 2

z k
40

w
2

w
2

dx0

(z 2 + x02 ) 2

dy 0 (z 2 + x02 ) 2

(z 2 + x02 + y 02 ) 2

Usando
el conveniente cambio:

02 + z 2 tg(u) = y 0
x

x02 + z 2 sec2 (u)du = dy 0


w
2

z k
=
40

z k
40

w
2

w
2

w
2

dx0

Z
3

(z 2 + x02 ) 2
dx0
2
(z + x02 )

((z 2 + x02 )sec2 (u)) 2

x02 + z 2 sec2 (u)du(z 2 + x02 ) 2

cos(u)du =

Por u
ltimo, con:
ztg() = x0
zsec2 ()d = dx0

z k
20

w
2

w
2

(z 2

dx0
+ x02 )

z k
=
20

w
arctg ( 2z
)

 w i
w
w
zsec2 ()d
k
k h

arctg

=
=
arctg
arctg
2
2
w
20
2z
2z
0
2z
arctg ( 2z
) z sec ()

Con el resultado obtenido, s


olo basta plantear la resta.
Problema 2
Dada la naturaleza de la geometra de la situacion, se hace necesario utilizar el ppio. de superposicion. Eso s, en
este caso podemos ocupar adicionalmente la Ley de Gauss, facilitando la obtencion del campo electrico. La idea
es primero utilizar lo anterior y encontrar con ello el valor de los campos en P, para luego sumarlos. (Para lo que
habr
a que efectuar una transformaci
on entre los sistemas de referencia).
Para la recta, utilizamos un cilindro que encierre un cierto vol
umen, con dicha recta en su eje. Denotamos A la
altura de el, informaci
on s
olo necesaria para caracterizarlo, prescindible en realidad. Despues, notamos posible
direcci
on y/o coordenada(s) de dependencia del campo, y con todo lo anterior se lleva a cabo el calculo. Dada la
simetra del problema, (en altura y orientaci
on se enfrenta el mismo escenario), es razonable suponer que el campo
depende del radio, y que su direcci
on apunta seg
un rho.
Z
QV


E dS =
0
S=dV

E = E()

Si bien la superficie S representa toda la cubierta del vol


umen del cilindro, solo ciertas partes son de interes. Las
tapas, arriba y abajo, tienen normales perpendiculares a , lo que muestra que los productos puntos en la integral
ser
an cero. De esta forma, s
olo importa el manto del cilindro. As:
Z A Z 2
A

ddz =
E()

E () =

2
0
0
0
0
Para el plano, efectuamos un razonamiento analogo. Por conveniencia, nuevamente elegimos un cilindro, que
ser
a quien encierre, eventualmente, toda la carga. Esta vez eso s, la simetra del problema cambia, por lo que las
suposiciones sobre dependencia y direcci
on del campo son igualmente distintas.
Sabemos que hay simetra en orientaci
on y radio, pero no necesariamente en altura. Por ello, podemos asumir
que el campo depender
a, en el peor caso, de z. Por la forma del problema, se sabe con certeza que la direcci
on
o -k,
dependiendo del sistema de referencia y la posicion en el espacio. Para esta situaci
del campo es seg
un k,
on
colocando correspondientemente el sist. de referencia. Adicionalmente vemos que solo las tapas
podemos asumir k,
del cilindro son de interes, pues el manto, con normal seg
un , anula su aporte a la integral. As, (ojo que el aporte
de las tapas es doble):
Z 2 Z
2
2
2

kdd

E(z)k
=
2 E (z) 2 =
E =
k

2
0
0
0
0
0
Para unir los resultados basta intercambiar por h en el campo producido por la recta, (para referir al punto
P directamente, ya que el aporte del campo generado por el plano es cte. independiente de la posicion), y escribir
en el sistema de referencia del plano. De un sencillo analisis geometrico,
cos(90 2)i sen(90 2)k =




E (P ) =
k+
cos(90 2)i sen(90 2)k
20
20 h
Por supuesto, queda a criterio de quien resuelva el problema el sistema en que quiera escribir la solucion, mientras
sea ortonormal y consistente en todo el termino.

Problema 3
(1) Planteando la integral, tendremos que:
Z
Z

( r)dV =
esf era
2

esf era

5k 2 2
sen()ddd = Qtotal
4a5

5k 4
5k
sen()ddd =
4a5
4a5

a5
5


(2) (2) = k

(2) Nuevamente por Ley de Gauss, asumiendo:

E = E()

Dada la distribuci
on de cargas, finita, debemos separar los casos para el campo electrico en . Usando coordenadas
esfericas, se tiene:
Si a
2

R
2

E()
sen()dd =
0

5k
4
4a5 sen()ddd

E()2 4 =

5k 5
k3

4 E () =

5
4a 50
4a5 0

Si > a
Z

Z
0

Qtotal

E ()dS =
0
0

E()
2 sen()dd =

k
k

E () =

0
42 0

Observaci
on: Es importante notar que el campo es contnuo en a. Esto no es algo trivial y en general debiera
analizarse por separado.
(3) El potencial electrico corresponde al valor del trabajo que se necesita para alejar o acercar una carga hacia otra
disposici
on de ellas. Como depende directamente del campo electrico, (dada su definicion), el calculo en esta parte
tambien estar
a diferenciado seg
un el radio de la esfera fuente. Dado que esta posee una cantidad finita de cargas,
se asume que V( = ) = 0. Por definici
on:

Z
r

V(r)=
E dr

r0
Si > a
Z

V () =

k
k
d

=
40 2
40

Observaci
on: Notar que en el resultado anterior la direccion del diferencial de posicion es en contra del vector
unitario , pues la partcula cargada es trada desde el infinito hasta una posicion a distancia radial de la esfera.
Si a
 4

Z a
Z
k
k3
k
k

a4

V () =

2
5
4a0
40 a5 4
4
40
a 4a 0
3

(4) La Ley de Gauss posee una forma en la que se expresa como sigue:

(
r)

E =
0
Con esto, es f
acil ver que lo pedido corresponde a los datos entregados. La divergencia del campo electrico es
nula fuera de la esfera, y vale la densidad de carga volumetrica dada al interior, dividida por la cte. 0 .
Problema 4
Ppto.
Se sugiere comenzar con la f
ormula que entrega la Ley de Gauss para situaciones con distribuciones volumetricas,
recien citada.

Enunciado Auxiliar No2 FI2A2


Prof. Aux: Felipe L. Benavides
Fecha: Lunes 11 de Agosto
Problema 1
Suponga que dos cargas puntuales en el vaco,q y + 12 q, se sit
uan en el origen y en el punto (a, 0, 0), respectivamente.
(Usando coordenadas cartesianas)
a) En que punto del eje x se anula el campo electrico?
b) Demuestre que la superficie equipotencial = 0 tiene forma esferica. Cuales son las coordenadas del centro de
dicha esfera?
Problema 2
Situaciones Aplicadas
V 
, dirigido hacia abajo.
a) El campo electrico en la atm
osfera sobre la superficie terrestre es aproximadamente 200 m
V 
, dirigido hacia
A 1400 [m] por encima de la superficie terrestre, el campo electrico de la atmosfera es de solo 20 m
abajo tambien. Cu
al es la densidad media de carga en la atmosfera por debajo de los 1400 [m]?
b) La resistencia
 V  dielectrica del aire, (es decir, el campo electrico lmite a partir del cual el aire llega a ser conductor)
es de 3 106 m
. Cu
al es el m
aximo potencial posible de un conductor esferico aislado, de 10 cm. de radio, en el
aire? (Potencial tal que no sobrepase la resistencia dielectrica del medio). Cual es el radio que debiera tener un
conductor esferico que, en las mismas condiciones anteriores de potencial maximo en el aire sin descarga, tuviese
una carga de 1 coulomb?
Problema 3
Considere una esfera de radio R en el vaco, con solo un hemisferio, (casquete o mitad), cargado, con densidad
de carga superficial constante . Calcule el campo electrico en el centro de la esfera. Hint: Calcule primero, por
definici
on, el potencial electrico en el eje OZ . No intente plantear las integrales del campo por definici
on.

Resoluci
on Auxiliar No2 FI2A2
Prof. Aux.: Felipe L. Benavides
Fecha: Lunes 11 de Agosto
Problema 1
a) Para esta parte, se escribir
an los campos electricos originados por cada carga, y con el ppio. de superposici
on,
se obtendr
a una expresi
on para el campo total, en tres dimensiones. Para cada caso utilizamos Ley de Gauss. Tal
y como antes, dada la geometra de las cargas se asume el campo radial seg
un coordenadas esfericas, y la superficie
encerrada corresponde al manto de la esfera de radio r. As:
Z
QV


E dS =
0
S=dV
Z

E(r)
r rr2 sen()dd =

E(r)
r rr2 sen()dd =

q
20

E (r) = 4q0 r2 r (1)



0
E (r ) = 8q0 r02 r0 (2)
Donde (1) corresponde al campo generado por la partcula con carga -q, y (2) a aquella con carga + 2q . Al pasar
el campo a coordenadas cartesianas, y usando el mismo sistema de referencia, centrado en -q, para ambos casos, la
superposici
on entrega, (trabajando s
olo la magnitud del campo):



1
1
|= q
| E (xi + yj + z k)

40 2 ((x a)2 + y 2 + z 2 ) x2 + y 2 + z 2

Al evaluar en el eje x, imponemos que y = z = 0. Agregando la condicion | E |= 0,

1
1
= 2 x2 = 2(x a)2 x2 4ax + 2a2 = 0 x = a(2 2) [m]
2
2(x a)
x
Lo que resuelve el problema.
b) Queremos probar que existe una superficie equipotencial nula con forma esferica, para lo que planteamos inicialmente el potencial generico. As,
q X
qk

V (
r)=

40
k
r
rkk
k

Entonces,

V (
r)=

q
40

"

1
1

2 ((x a)2 + y 2 + z 2 ) 2

1
1

(x2 + y 2 + z 2 ) 2

Es importante notar en esta expresi


on que se asume que la referencia es tal que el potencial se anula en el infinito.
Esto es no s
olo razonable particularmente, sino que se extiende a cualquier distribucion finita de cargas. Eso s,
pudiera ser que el potencial fuera nulo en otra(s) posicion(es) del espacio, pero a
un si esto sucede, conviene usar lo
antes nombrado. De no hacerlo as, la expresi
on inicial generica no es valida, pues falta un termino constante que
dependera de la referencia usada. Por lo anterior, tiene sentido usar esta expresion para buscar una superficie a
potencial cero con forma, a
un sabiendo que en el infinito existe una, tambien nula. Imponiendo V = 0,
2 (x a)2 + y 2 + z 2

 12

= x2 + y 2 + z 2

 21


x2 + y 2 + z 2
= (x a)2 + y 2 + z 2
4

Al resolver, las soluciones quedan:


(
(x
(x

4a 2
3 )
4a 2
5 )

+ y 2 + z 2 = 4a9
2
+ y 2 + z 2 = 4a
25

(+)
()

Con esto, las coordenadas del centro de la esfera obtenida en (+), (pues (-) no tiene significado fsico), son:
,
( 4a
3 0, 0).
Problema 2


Adicionalmente, se supone que la densidad de
a) Por la simetra de la situaci
on, se asume que E (
r ) = E(z)(k).
carga libre de la atm
osfera es constante, (a
un cuando no lo sea en esta situacion, pues para efectos de lo buscado es
equivalente), y que esta se comporta como el vaco en terminos electricos. Planteando la ley de gauss en su forma
diferencial, en coordenadas cartesianas, colocando el origen sobre la superficie terrestre,

(
r)
E

E =

(z) =
0
z
0
El signo menos aparece pues las coordenadas cartesianas tienen sus vectores unitarios definidos positivamente,
(i.e., con orientaci
on hacia afuera del origen), luego el - que aparece en el campo inicial debe trasladarse a su
magnitud. Esto es adem
as coherente, pues los datos confirman que la magnitud del campo disminuye con la altura.
As,
Z z

dz = C z
E(z) =

0
z0 0
Con C constante. Evaluando los datos,
(
V 
E(0) = 200 m

1800
 V  E(z) = 200 z 20 = 200 1400 =
0
0
1400
E(1400) = 20 m




F
Tomando 0 ' 8.854 1012 m
, ' 1.14 1012 mC3
b) Para esta parte, primero se obtiene la carga que debe tener la esfera conductora para alcanzar el campo crtico
justo en la vecindad del manto. Con ello, se calculara el potencial. As, (en coordenadas esfericas),
Z 2 Z


Q
Q

E ( r ) = E(r)
r
E(r)
r rr2 sen()dd =
r
E (
r)=
2

4
0
0r
0
0
Evaluando, se obtiene la siguiente expresi
on para despejar Q:
3 106 =

Q
40 0.12

El potencial crtico m
aximo queda:
0.1

3 106 40 0.01

r (r)dr
= 300000 [V ]
40 r2

De forma an
aloga, se resuelve la situaci
on con una esfera conductora de 1 coulomb. Se tendra, (con el mismo
valor antes usado de 0 ):
Z

3 106 =

1
r ' 54.73 [m]
40 r2

Problema 3
Primero es importante notar que es posible calcular el campo electrico en el origen por definicion y dicho c
alculo
es mucho m
as corto que el presentado a continuacion. Queda propuesto, puesto que la solucion por este camino es
algo m
as compleja de obtener, dado que requiere argumentacion geometrica mas directa. Para obtener el campo
electrico, se obtendr
a primero el potencial electrico, y luego se calculara el gradiente con lo que se tendra lo pedido.
Por definici
on, (con referencia de potencial nulo en el infinito),
Z
1
1

V(r)=
dq(
r 0)

40
k r
r0k

con la integral definida en este caso en la superficie del casquete, cargada.


r representa la(s) posici
on(es)

0
deseada(s), y r la(s) ubicaci
on(es) de la(s) carga(s). Por conveniencia se centra el casquete en el origen. Como
se busca eventualmente tomar gradiente, no tiene sentido calcular el potencial en un punto. Se calcular
a, en vez,
(en z), y luego se impodra z=0. (La idea de calcular potencial en cierto eje para
el potencial en todo el eje k,
luego tomar gradiente, imponer cierta(s) condicion(es) y finalmente obtener el campo electrico deseado es utilizada
a menudo). Para escribir el termino en el denominador de la fraccion dentro de la integral, se observa que:

k
r
r 0 k=k z k r
r k=k z k rsencosi rsensenj rcosk k
=

z 2 2zrcos + r2

El casquete esferico queda parametrizado por:

r = R

0 2
De esta forma, la expresi
on queda: (con dq = dS)
Z
R2 sendd
2R2
send

=
=
2 2zRcos + R2
2

4
z
z

2zRcos
+ R2
0
0
2
2


i
p
R2
R h
2 p 2
2
=
z 2zRcos + R
=
z + R z 2 + R2

20
2zR
20 z

V (z) =

1
40

Usando ahora,

E = V
se confirma con la ecuaci
on que el campo electrico, por simetra, en el eje z depende solo de z, (y dada la
Calculando, tenemos:
disposici
on de cargas apunta seg
un k).
!
R

R2 1 R2 +z2
E (z) =
k
20
z2

Se desea el campo E (z = 0), pero la expresion anterior no esta bien definida all. Sin embargo, como al tender z
a cero, tanto numerador como denominador caen a cero, es posible utilizar la regla de lHopital. As, (y esto finaliza
el problema):


d
 
R
dz 1 R2 +z 2

R2
V
E (z = 0) = lim E (z) =
k lim
=
k
d
2
z0
20 z0
40
m
dz (z )

Enunciado Auxiliar No3 FI2A2


Prof. Aux.: Felipe L. Benavides.
Fecha: Lunes 18 de Agosto de 2008
Problema 1
El espacio entre dos superficies esfericas concentricas conductoras, (con cargas Q y Q y radios a y b respectivamente,
a < b), est
a lleno con dos materiales dielectricos caracterizados por 1 y 2 respectivamente. Estos dos materiales
est
an separados por un plano ecuatorial. Suponga que los campos son proporcionales a r.
a) Obtenga la densidad de carga libre en cada una de las cuatro superficies semiesfericas.
b) Encuentre los valores de desplazamiento electrico que caracterizan la esfera.
c) Obtenga la diferencia de potencial entre los cascarones esfericos. Aparecen cargas de polarizacion? D
onde?
C
omo las calculara?
Problema 2
Se desea medir la altura h de un lquido, en un recipiente extenso de forma rectangular hecho de vidrio. Para ello, se
colocan tres placas conductoras paralelas rectangulares planas. Entre las placas superior e inferior se establece una
diferencia de potencial V0 mediante una batera. Si la permitividad del lquido es y la segunda placa conductora se
encuentra descargada, determine el potencial de esta en funcion de h. Suponga que el ambiente tiene permitividad
0 . (Ver figura 1)

Figura No 1
Problema 3

Asuma la energa potencial de interacci


on entre dos dipolos, (el sistema que constituyen), ubicados en
r1 y
r2 ,
conocida, e igual a



1
p1
p2
(
p1
r 21 ) (
p2
r 21 )
U=
3

40 k
r 21 k3
k
r 21 k5

Con
r 21
r2
r 1 . Utilice este resultado para resolver la siguiente situacion particular:
Dos moleculas de agua, (de momento dipolar p cada una), se encuentran separadas una distancia D. Considere
las siguientes situaciones ((i) y (ii)). En (i), los dos momentos dipolares apuntan en direccion perpendicular a la
lnea que une sus centros. En (ii), los momentos dipolares son colineales con dicha recta. En cada caso los dipolos
apuntan en la misma direcci
on. Dado lo anterior, obtenga:
a) Diferencia de energa entre las posiciones i) y ii).
b) Si se les libera desde el reposo, En que direccion se moveran las moleculas en cada caso? Puede estimarse la
aceleraci
on, velocidad y tiempo de interacci
on? c) Obtenga la expresion de la energa de un sistema compuesto
por dos dipolos. Puede ser u
til partir de la expresion del potencial de un dipolo electrico. (Propuesto).

Resoluci
on Auxiliar No 3
Prof. Aux.: Felipe L. Benavides
Fecha: Lunes 18 de Agosto de 2008
Problema 1
El sistema queda representado por la siguiente figura,

Figura No 1
a) Como las superficies esfericas son conductores, las cargas presentes por enunciado +Q y Q son, necesariamente, libres. As,
a 4a2 = Q y b 4b2 = Q
De all, pueden deducirse los valores de las densidades de carga libre presentes, donde se asumen distribuciones
de carga uniformes pues se trata de materiales conductores (lo que permite escribir que las cargas por unidad de
area, que multiplicadas por el

area, ser
an la carga total). La mitad de cada cascaron, tendra,
(
Q
Q
2
Sup. r = a
4a2 2a = 2
Q
Q
2
4b2 2b = 2 Sup. r = b
b) Para calcular el vector Desplazamiento Electrico, utilizamos ley de Gauss en su forma general para materiales
polares. No podemos utilizar la ley de Gauss en su forma basica para el campo electrico pues esta asume que
se encierran cargas en una superficie no polar, el vaco. De argumentos geometricos, y usando que los campos
apuntan seg
un r, se obtiene que dentro de la esfera, dentro del primer cascaron conductor esferico, tanto campo
como desplazamiento electrico son nulos.
Como se trata de caracterizar la esfera, basta escribir el valor del Desplazamiento entre los cascarones conductores
en cuesti
on. Al encerrar con una superficie esferica el cascaron conductor de radio a, con una esfera de radio r,
(a < r < b), usamos, en coordenadas esfericas, que

D(r) = D(r)
r
As,


D dS = QL

S=dV
2 Z

Z
0

D(r)
r rr2 sendd = QL

Aqu debemos detenernos un poco. Como se trata de dos materiales diferentes, (con permitividades dielectricas
diferentes), en principio no es posible asumir que se trata solo de un vector desplazamiento electrico. Perfectamente
podran haber dos, diferentes, en cada material. La u
nica forma de despejar esta interrogante es usar condiciones

de borde. Estas,
(que se cumplen en cualquier situacion), son;
(
E1t = E2t
(i)
D1n D2n = l (ii)

Figura No 2

Suponiendo dos materiales seg


un la figura 1, y campos D y E , (i) significa que la componente de campo
electrico tangencial a la superficie es equivalente entre ambos materiales, y (ii), que las componentes normales del
desplazamiento entre ambos materiales difieren seg
un la carga libre que haya en la interfaz, seg
un se muestra en la

figura 2. Usando (i), D = E (= r 0 E ), y evaluando la expresion en la region ecuatorial del sistema,


D1
D2
=
1
2
Volviendo a la ecuaci
on de Gauss, tendremos,
D1 (r) +

2
Q
D1 (r) =
1
2r2

As,

D 1 (r) =

Q
2r2

D 2 (r) =

Q
2r2

1
1 + 2

2
1 + 2

r
r

c) Para la diferencia de potencial, planteamos la resta de potenciales entre a y b. (Intuitivamente, el potencial


es mayor en la carga positiva, i.e., en a, y menor en b).
Z

V (a) V (b) =


E d
r


E d
r =

De las ecuaciones anteriores,

E (r) =

Q
1
r
2r2 (1 + 2 )
2


E d
r =

Z
a


E d
r

Finalmente,
V (a) V (b) =

Q
2(1 + 2 )

1 1

a b


=

Q
2(1 + 2 )

ba
ab

Efectivamente, por la presencia de cargas libres, hay cargas de polarizacion en el sistema. Dichas cargas se alojan
en la vecindad de los casquetes esfericos y tienen signo opuesto a ellos, mayoritariamente. Pueden ser calculadas
obteniendo el vector polarizaci
on y con el, las cargas superficiales y volumetricas de polarizacion.
Problema 2
Lo primero en este problema es comprender porque es posible obtener una expresion para el potencial en funci
on de
la altura de cierto lquido, y de all calcular dicho potencial. La idea de la situacion es que el potencial en la placa
conductora a altura h depende de los valores del campo electrico que estan definidos bajo ella. Como el potencial
es cero en la placa inferior, (pues est
a conectado a tierra), la medicion de los campos comienza desde all, y se ve
afectada por los materiales que atraviesa.
La batera inicialmente coloca una diferencia de potencial V0 entre las placas superior e inferior. Dicho potencial

est
a siendo generado por un campo electrico que apunta desde las cargas positivas, a las negativas, i.e., seg
un k.
Es intuitivo pensar que si hay un material con permitividad dielectrica en el camino del campo electrico, dicho
campo se ver
a disminudo.

Esto
pues el campo atraer
a las cargas negativas hacia arriba, y las positivas, hacia abajo, polarizandolo, y por
lo tanto formando un campo electrico por cargas de polarizacion opuesto al inicial, dentro del material. De esta
forma, la altura del frasco dir
a acerca de la cantidad de lquido que obstruira el paso del campo, y por lo tanto,
el potencial en la segunda placa conductora descargada variara seg
un h.
Necesitamos entonces conocer el campo, e integrarlo despues. Para ello, planteamos ley de Gauss para materiales
polares, con la carga -Q entre medio, y mediante condiciones de borde despejamos el valor de campo afuera. Lo
primero es notar que el campo electrico arriba de la primera placa conductora es cero, y bajo la tercera, tambien.

Esto
pues el potencial no se ve alterado ni mas arriba ni mas abajo de las placas, luego el campo debe ser nulo.
Usando un cilindro de radio r con altura y disposicion cualquiera tq. se encuentre dentro del material su tapa
superior, y bajo la placa conductora inferior la tapa opuesta, se tendra, (asumiendo campos y desplazamiento a lo

m
as dependientes de z, y apuntando seg
un -k),

D(r) = D(z)k
I


D dS = QL

S=dV

S
olo las tapas superior e inferior constituyen aportes, pues el manto tiene normal perpendicular al desplazamiento
electrico. La tapa de arriba dentro del material entrega un valor, pero la que se encuentra bajo la placa conductora
inferior no, pues notamos ya que el campo electrico era nulo. En coordenadas cilndricas,
Z

dd

D(z)dd =
0

Dr2 = r2 D =
As, con la magnitud del desplazamiento electrico obtenida,

D(r) = k

E (r) = k

Dentro del material. Afuera, podemos usar ley de gauss denuevo, pero para la placa superior esta vez. La otra
opci
on, es simplemente usar condiciones de borde. Como la interfaz, definida como la capa que separa el lquido del
3

aire, no est
a cargada, los vectores desplazamiento electrico son iguales, y con ello,(D1n D2n = l D1 = D2 ),
afuera del material,

E = k
0
Con los campos listos, basta despejar la expresion para el potencial en h. El u
nico problema es que es
inicialmente desconocido, (no es dato del problema), aunque es calculable. De plantear que el potencial entre las
placas superior e inferior es V0 , puede despejarse dicha constante. As, (distinguiendo con n
umeros los campos para
destacar que no son iguales),
h

Z
V0 =


E 1 dr +

h+d


E 2 dr +

d+a


h (d h) a
+
+
E 2 dr =

0
0

V0 0
()
h0 + (d h + a)

As,
Z
V (h) =


E 1 dr +

E 2 dr = h + (d h) ()

Reemplazando (*) en (**),



V (h) = V0

h0 + (d h)
h0 + (d h + a)

Problema 3
Planteando las situaciones descritas y la energa asociada seg
un la formula,
i.

r 21 =
r2
r 1 = D
x,
p1 =
p 2 = p
y
Ui =

ii.

1 p2
40 D3

r 21 =
r2
r 1 = D
x,
p1 =
p 2 = p
x
Uii =

1
40

p2
p2 D 2
3 5
3
D
D


=

p2
20 D3

As,
Ui Uii =

3 p2
40 D3

b) Si se liberan desde el reposo, la energa tendera a disminur, luego las distancias entre sus centros ser
an
mayores o menores seg
un las ecuaciones muestren. En i, la energa disminuye si D aumenta, por lo que los dipolos
se repeler
an. En ii, la energa disminuye si D disminuye, por lo tanto, los dipolos se atraeran. En efecto, puede
estimarse aceleraci
on, velocidad y tiempo de interaccion, por ejemplo, modelando como un sistema newtoniano, y
utilizando que
k Fi k=k

Ui
Uii
k, k Fii k=k
k
D
D

Enunciado Auxiliar No4 FI2A2


Prof. Aux.: Felipe L. Benavides
Fecha: Lunes 25 de Agosto de 2008
Problema 1
En el centro de una cavidad esferica de radio a practicada en un bloque de material dielectrico de permitividad
relativa k, se coloca una carga puntual q. Calcule el potencial electrico en todos los puntos del espacio. Demuestre
adem
as que la suma de las cargas inducidas y la carga original es kq , independiente de a.
Problema 2
Se tienen dos planos conductores paralelos infinitos de ecuaciones x = d, y x = d, respectivamente. El plano
x = d est
a conectado directamente a tierra y el otro, a traves de una batera cuya diferencia de potencial entre
sus bornes es V0 . La regi
on comprendida entre los planos esta rellena con un material dielectrico, cuya constante
dielectrica es:
= (x) =

4

x 2
d

+1

Calcule:
a) Campo electrico en todo el espacio, en funcion de la densidad de carga de los planos.
b) Funci
on potencial en todo el espacio.
c) Densidad de carga superficial en cada uno de los planos en funcion de V0 .

b) Vector polarizaci
on, P .
c) Densidad de carga de polarizaci
on, p , y densidad de carga superficial de polarizacion, p , en las dos superficies
del dielectrico.
Problema 3, Propuesto
Una esfera de radio R de material con permitividad dielectrica 0 esta cargada con densidad de carga uniforme, de
tal modo que su carga total es q.
a) Calcule la energa del sistema.
b) Suponga que la esfera se trata de un electron. La energa de este, obtenida seg
un consideraciones relativistas,
es del orden de 1013 Joule. Obtenga el orden de magnitud del radio del electron.

Resoluci
on Auxiliar No 4 FI2A2
Prof. Auxiliar: F. L. Benavides
Fecha: Lunes 25 de Agosto de 2008
Problema 1
Para calcular el potencial electrico en todo el espacio es necesario obtener el campo electrico para todas las regiones
del sistema. Poniendo el origen en la posici
on de la carga q, por casos, se obtiene: (Ojo que el dato es k permitividad
relativa. Ella es tal que 0 k = permitividad dielectrica del material).
r a;
2

Z
0

E(r)r2 sen()dd =

E (r) =

q
r
40 r2

r > a;

D(r) =

q
q
r,
E (r) =
r
4r2
40 kr2

El potencial electrico queda: (con la referencia de potencial nulo en el infinito)


E dr

Si r > a,

V (r) =

 r
q
q
1
q
dr =
=
2
4r 0 k
40 k r
40 kr

Si r a,

V (r) =


E dr =


E dr


E dr =

q
(dr)
40 r2

Z
a

q
(dr)
4k0 r2



1
1
1

+ +
ak a kr

q
V (r) =
40

Eso por un lado. Para demostrar el resultado que se pide en relacion a las cargas inducidas, primero es necesario

obtener el vector de polarizaci


on. De la definicion, P = ( 0 ) E , se llega a:

P (r) =

q(k1)

4kr 2 r

r>a
ra

De aqu que:

1
p = P = 2 (P r2 ) = 0
r r

h
i

p = P n

r=a

q(k 1)
4ka2

De esta forma, al sumar la carga total de polarizacion superficial junto con la puntual en el origen, vemos que:

q
q(k 1)
+q =
k
k

Es decir, la carga total no depende del radio, tal y como se deseaba probar.
Problema 2
Primero notamos que el campo electrico para |x| > d es nulo. Esto pues, dada la descripcion del sistema, la diferencia
de potencial entre cualquier par de puntos al lado izquierdo o derecho del eje es cero, i.e., no puede haber campo.
En general, cuando hay planos infinitos se asume este resultado, que no depende del tipo de dielectrico que haya
entre medio. Puede probarse m
as detalladamente calculando los efectos producidos por cada conductor cargado, y
luego sumando. La idea te
orica detr
as de esto es que una lnea de campo nunca es abierta, todas parten y terminan
en cargas, (positivas y negativas respectivamente). Si hubiese lneas de campo hacia la derecha o izquierda, tendran
que devolverse en el infinito, pero dado que son rectas, (pues los conductores son infinitos), esto no puede ocurrir.
a) Por ley de Gauss, utilizando una superficie cerrada cilndrica entre el medio de la derecha, la placa conductora
con carga +, (x = d), y el medio en cuesti
on, solo influye la cara del cilindro que queda encerrada en x < d. (El
manto tiene normal perpendicular y el campo para x > d es nulo). Entonces,
Z


D dS =

dS D4S = 4S D = i

E (x) =

40


x 2
d

i
+ 1 i |x| < d
|x| > d

b) Para la funci
on potencial, calcularemos primero dentro del dielectrico, y luego concluiremos para todo el espacio.
As,
|x| < d,

V (x) =
d



  3


 x 2

1 x + d3
+ 1 dx =
+
(x
+
d)
40
d
40 d2
3

V (x) =

h  3 3
i
1
x +d

+ (x + d)
40 d2
3

|x| < d

V0

x>d
x<d

c) Las densidades de carga superficial son iguales en magnitud y opuestas en signo. Basta que igualemos el potencial
en x = d con las dos expresiones que se tienen, y se despeja la densidad superficial. Ojo, que el campo no est
a
definido para x = d, no as el potencial, que es la integral del primero. Como la discontinuidad es finita, no hay
complicaciones.


  3

3V0 0
1 d + d3
+ (d + d) = V0 =
2
40 d
3
2d

(Para basta cambiar el signo).


d) El vector polarizaci
on queda de la siguiente forma:

P = ( 0 ) E

P (x) =


x 2
d

+1

|x| < d
|x| > d

e) Las densidades de carga de polarizaci


on quedan de la siguiente forma:

x
p = P = 2
2d

p = P n
=

1
1

2 0

Donde el p calculado corresponde al efecto a la derecha. (A la izquierda es simplemente el mismo, con signo
contrario, pues las normales son opuestas.
Problema 3

Este
problema queda propuesto puesto que es simplemente ilustrativo respecto de las aplicaciones de la teora
estudiada a la realidad. Para el c
alculo de la energa del electron, se propone simplemente realizar:
1
E=
2

E D dV

Donde los vectores campo y desplazamiento electrico son muy sencillos de calcular, para la carga q dada. Luego,
la idea es comparar el valor seg
un consideraciones relativistas, teoricas, de 1013 [J], para un electron, y con ello
estimar el orden de magnitud de su radio.

Enunciado Auxiliar No5 FI2A2


Prof. Auxiliar: Felipe L. Benavides
Fecha: Lunes 1 de Septiembre de 2008
Problema 1
Considere un sistema infinito de simetra cilndrica compuesto, al centro, por un alambre rectilneo con densidad de
carga 0 uniforme, rodeado de un cilindro de radio a, de material dielectrico con constante dielectrica a , a su vez
rodeado de un cilindro conductor de radio exterior c el cual, finalmente, esta rodeado de un cilindro dielectrico de
radio exterior b y de constante dielectrico b . El cilindro conductor esta cargado; su carga por unidad de longitud
es 1 . (Cada anillo). Determine el campo electrico y de desplazamiento en todas partes.
Problema 2
Considere 2 casquetes esfericos concentricos conductores de radios a y b, con a < b. El conductor de radio a est
aa
un potencial V0 , y el conductor de radio b est
a conectado a tierra. (Potencial cero). El espacio entre los conductores
se rellena con un dielectrico de permitividad , seg
un la figura 1. Calcule el campo y desplazamiento electrico,
vector polarizaci
on, densidades de carga de polarizacion superficial y volumetrica, entre los casquetes. (La carga Q
es desconocida).

Figura No 1.
Problema 3
-CondensadoresSituaci
on I. La permitividad de cierto material vara linealmente de una placa a otra en un condensador de placas
paralelas. Si 1 y 2 son los valores de la permitividad de cada una de las placas, y 1 < 2 , determine la capacidad
Q
por unidad de
area, siendo d la separaci
on de las placas. (Recuerde: capacidad C = 4V
).
Situaci
on II. Considere un sistema de electrodos rectangulares de area unitaria, al cual se le aplica una diferencia
de potencial V0 . Entre las placas se coloca una lamina conductora ideal. Determina las densidades de carga libre
que aparecen en las caras de la l
amina central.
1

Soluci
on Auxiliar No 5 FI2A2
Prof. Aux.: Felipe L. Benavides
Fecha: Lunes 01 de Septiembre de 2008
Problema 1

Este
problema es ciertamente sencillo. Solo es importante tener cuidado con los efectos en los
bordes. Dada la geometra del sistema, podemos asumir que el campo tiene simetra cilndrica, con
dependencia s
olo en , y que apunta seg
un . Con lo anterior, en cada caso basta plantear Gauss
con campo o desplazamiento electrico, (seg
un corresponda), y concluir. Se tendra que:
< a;
z0

z0

0 dz

D()ddz =
0

0
0
, E () =

D() =
2
2a

a < < c;

D() = E () = 0

c < < b;
z0

z0

z0

0 dz + (c a )

D()ddz =
0

1 dz
0

0 + 1 (c2 a2 )
0 + 1 (c2 a2 )
D() =
, E () =

2
2b

> b;

z0

R z0

E()ddz =
0

0 dz + (c2 a2 )
0

R z0
0

1 dz

0 + 1 (c2 a2 )
0 + 1 (c2 a2 )
D() =
, E () =

2
20

= a, c;

D()@, E ()@
Es importante notar que se tuvo cuidado con los bordes de cada vol
umen, en el caso del vol
umen
conductor. Como el campo es cero justo dentro de el, dadas sus propiedades, (reacomodo de cargas
frente a un campo electrico cualquiera), y distinto de cero justo despues, no es posible definirlo en
la interfaz. Observamos tambien que en = b el vector desplazamiento es contnuo, pero el campo
no. Esto se debe a que en la superficie de separacion entre el vaco y el dieletrico, aparece una
densidad de carga de polarizaci
on, no carga libre. (Lo que es coherente con la condicion de borde
D1n D2n = s )
Problema 2
Para resolver este problema, se utilizara gauss nuevamente, asumiendo que dada la geometra del

sistema, el campo depende s


olo del radio, en coordenadas esfericas, y apunta seg
un r. Esta
suposicion
es algo fuerte, pero puede fundamentarse pues el dielectrico es coherente con la esfera en terminos
de que la superficie tangencial, entre los casquetes conductores, se orienta seg
un r. As, llamando
I al vol
umen dentro del dielectrico, y II al restante entre los casquetes, basta despejar el valor del
campo y usar las f
ormulas conocidas para todo lo demas pedido. Queda: (entre los casquetes)
a < r < b;

Z
0

D1 (r)
0

D(r)r2 sen()dd = Q

r2 sen()dd + D2 (r)

r2 sen()dd = Q

D1 (r)2r2 (1 cos()) + D2 (r)2r2 (1 + cos()) = Q

En este punto debemos usar alguna ecuacion que relacione ambos desplazamientos. De las condiciones de borde, se tiene que:

D1n D2n = s
E1t = E2t

(1)
(2)

Ambas condiciones son, en general, linealmente independientes. Sin embargo, dado que D = E ,
con conocido en ambos lados, necesitamos una sola de ambas. Utilizaremos la expresion (2) por
simplicidad, pero la primera, m
as la definicion del vector desplazamiento, tambien resolveran el
problema. (S
olo que habra que usar consideraciones entre el vol
umen dentro de los casquetes,
y aquel afuera, o dentro del casquete conductor peque
no, radio a. Usando lo recien nombrado,
tenemos:
D2
D1
=

D 1 (r) =

Q
2r2 1 cos() +

0
(1

E (r) =

+ cos())

 r, D 2 (r) =

Q
2r2

1 + cos() +

0 (1

cos())

 r y

Q
r
2r2 (0 (1 + cos()) + (1 cos()))

Notamos a esta altura que la carga Q, seg


un el enunciado, es desconocida. Esto no es una complicaci
on, pues con la expresi
on recien obtenida del campo, basta calcular el potencial e igualar con
el dato V0 del problema. Queda: (usando que g0 = 0 (1 + cos()) + (1 cos()))

r0

E dr = V (
r)

Q
dr = V0
2r2 g0

Q=

2g0 V0 ab
ba

Para la polarizaci
on, separamos por casos:
[0, ], [0, 2] y r (a, b);

( 0 )Q
r
P =
2r2 g0

[, ], [0, 2] y r (a, b);

P = (0 0 ) E = 0

Las densidades de carga de polarizaci


on quedan: (Para el primer caso con P no nulo)

1
p = P = 2 (P r2 ) = 0
r r

p =

( ( )
2a2 g00
(0 )
2b2 g0

r=a
r=b

Problema 3
Situaci
on I.
Primero planteamos la expresi
on lineal para la permitividad del dielectrico. Se tiene:
(z) =

2 1
z + 1
d

Buscamos
C=

Q
,
4V

pues con esta expresi


on, la capacidad por unidad de area es simplemente lo mismo sobre el
rea total. (Asumiremos conocido Q magnitud de la carga de las placas conductoras del capacitor,
a
y calcularemos respecto del este valor algo del estilo 4V (Q), para despues simplemente dividir.
Usando Gauss en el electrodo superior con una superficie cerrada cilndrica, imponiendo que el
campo arriba de la placa conductora es nulo y que apunta hacia abajo, queda:
Z

D dS = Q

Q
D4S = Q D =
k E (z) =
4S
4S

Q
2 1
d z

+ 1

 k

Integrando el campo para obtener el potencial,


Z

Q

dz = V0
+ 1 4S

2 1
d z

Q
4S(2 1 )
 
=C=
4V
dln 21
Por lo tanto la capacidad por unidad de area pedida es:
C=

Situaci
on

(2 1 )
 
dln 21

II.

Queda propuesto. La versi


on inicial es simplemente demostrativa, la idea es que se pruebe con
distintas configuraciones, para distancias diferentes entre la lamina conductora y las placas superior
e inferior, y para diferentes permitividades dielectricas.

Enunciado Auxiliar No6 FI2A2


Prof. Aux.: Felipe L. Benavides
Fecha: Lunes 22 de Septiembre de 2008
Problema 1
Ley de Ohm y Resistencias
Para la figura 1, asumiendo que se trata de cascarones esfericos concentricos, y suponiendo conocidos los datos
all presentados, (salvo V0 que es innecesario para el resultado final), cual es el valor de la resistencia R entre los
conductores?

Figura No 1
Problema 2
Ley de Ohm, Resistencias y Leyes de Kirchoff
La llamada protecci
on diferencial de los sistemas de suministro electrico domiciliarios, consiste en un interruptor
autom
atico que funciona desconectando el circuito cuando la corriente que entra es diferente de la corriente que
sale. Supongamos que se tiene conectada una estufa electrica de una potencia 2[kW ], que tiene un defecto tal que
el punto medio de la resistencia calefactora esta conectado a la caja metalica de la estufa. Si una persona toca la
caja, y adem
as est
a conectada a tierra a traves de sus pies, va a pasar una corriente por la persona. Se pide calcular
cual es la diferencia de corriente entre los dos cables de alimentacion de la estufa, sabiendo que la persona presenta
una resistencia entre una mano y sus pies, de 10[k].

Figura No 2
1

Problema 3
n de Continuidad
Ecuacio
Considere un condensador de placas planas paralelas cuyo espacio interno se llena con dos bloques de materiales
distintos, cuyas permitividades y conductividades valen respectivamente (1 , g1 ), y (2 , g2 ), como se indica en la
figura 3.
a) Si entre las placas se establece una diferencia de potencial V0 , calcule la densidad de cargas libres superficiales
en la regi
on interfacial en regimen permanente.
b) Suponga ahora que, despues de haberse establecido el regimen permanente, se desconecta la batera y cada
placa se conecta a tierra. Determine la evolucion temporal de la densidad de carga en la region interfacial.

Figura No 3
Problema 4
Fuerza de Lorentz
El diagrama de la figura 4 representa un dispositivo para medir las masas de iones. Un ion de masa m y carga +q
sale de la fuente F pr
acticamente en reposo. Luego, el ion es acelerado por una diferencia de potencial V y se le

permite entrar a una regi


on de campo magnetico de modulo | B |. (El campo sale de la hoja). En presencia de
este campo se mueve en un semicrculo, incidiendo sobre una placa fotografica a una distancia x desde la rejilla de
entrada. Obtenga expresiones, en terminos de B, q, V y x, para: la masa m del ion, el tiempo que se encuentra
dentro del sistema y la posici
on en funci
on del tiempo.

Figura No 4

Resoluci
on Auxiliar No6 FI2A2
Prof. Aux.: Felipe L. Benavides
Fecha: Lunes 22 de Septiembre de 2008
Problema 1

Este
tipo de problemas es muy com
un, y su solucion por tanto tambien es bastante estandar. La idea es calcular la
corriente en funci
on del potencial V0 al que esta sometido el material, y de all simplemente despejar la fracci
on VI0 .

Se asume por supuesto I constante, no as J , que depende de la posicion para permitir que lo anterior se cumpla,

(se trata de regimen permanente). Con ella entonces, despues de despejar J , se obtiene E , y con el, V0 . Dada la
disposici
on geometrica del sistema, el vector densidad de corriente apunta seg
un r. Resolviendo entonces,
Z
Z 2 Z

I=
J dS =
J(r)r2 sen()dd
0

I = 4J(r)r2 J (r) =

I
r
4r2

Ahora, por ley de Ohm,

gE = J E =

I
r
4gi r2

Con i=1,2. Como la densidad de corriente es contnua en el espacio, (ppto. probarlo en este caso, aunque es muy
sencillo), la expresi
on se mantiene y s
olo varan los g de la conductividad de los materiales. Calculando entonces la
diferencia de potencial, se obtiene:
V0 = V (a) =
Z
=
c

Z c
I
I
dr +
dr
2
2
4g
r
4g
1
2r
b
a
a




I
1 1
I
1 1
V0 =

4g1 a b
4g2 b
c




V0
1
1 1
1
1 1
R=
=

I
4g1 a b
4g2 b
c


E d
r =


E d
r =

Problema 2
Para resolver este problema se plantea un circuito equivalente donde se divide la resistencia del calefactor en 2
trozos, y se conecta, en paralelo con la de m
as abajo, una equivalente a la persona. Por notacion, usando que I1 es
la corriente que circula por el primer trozo de la resistencia, I2 el que circula por la persona, e I3 , por el segundo
trozo, se pide conocer entonces:
I2 = I1 I3
Para ello, se plantean las siguientes ecuaciones, (de Ohm y Kirchoff):
(LV K) 220 =

I1 R I3 R
+
2
2

(LV K)

I3 R
= I2 10000
2

(LCK) I1 = I2 + I3
Para despejar el sistema falta una ecuaci
on, pues R es desconocido. Aqu usaremos que:
V2
= 2000 ()
R
Es decir, planteamos la expresi
on de potencia para el calefactor. Seg
un el enunciado, podemos usar que la
potencia consumida es nominal, y no es aquella referida al momento de la falla. El calculo utilizado para esto
es exacto, y la resoluci
on tiene total precisi
on. Si la potencia referida en el enunciado se tratase del consumo al
momento de producirse la falla, la u
ltima expresion es solo una aproximacion. Dado que la corriente que circula
por la persona es peque
na, (pues la resistencia es muy grande), no es una mala aproximacion. Debieran sumarse
todos los otros terminos, pero s
olo consideran corrientes peque
nas elevadas al cuadrado, i.e., valores ciertamente
despreciables frente a la primera componente. Por ello, el procedimiento es equivalente en ambos casos, salvo que en
el primero es exacto y en el segundo es una aproximacion. (Aunque razonable. Sumar todos los terminos tambien
resuelve el problema, s
olo que el despeje del sistema es muy largo y complejo). Resolviendo, tendremos:
I2 =

220 2
R + 4 104

De (*), R = 24.2[]
I2
= 0.011 = 11[mA]
El valor confirma que la aproximaci
on no es mala, pues efectivamente I2 es peque
na, dado que la resistencia del
calefactor es mucho menor que la de la persona. (En ordenes de magnitud incluso).
Problema 3
a) Para encontrar la densidad de carga libre en la interfaz, podemos usar

( D 2 D 1) n
= l (1)
En regimen permanente, dado que los materiales exhiben una conductividad no nula, hay corriente. Por continuidad, (conservaci
on de la carga):


J = (
r , t)
t
La condici
on de regimen permanente queda:

(
r , t) = 0
t
As

J (z) = J k
Tomando positivo z de izquierda a derecha, seg
un el dibujo de la figura en el enunciado. Como adicionalmente,
(por ley de Ohm):

J = gE

J
J
E 1 = k;
E 2 = k
g1
g2

J
J
D 1 = 1 k;
D 2 = 2 k
g1
g2
2

Pero a priori no es conocida ni la corriente, ni su densidad. Por ello, necesitamos encontrar una expresi
on que
ligue los datos restantes del problema con la informacion ya construda. En efecto, por potencial, tendremos:
Z

d1 +d2


E d
r = V0

E1 d1 E2 d2 = V0
J
J
d1 + d2 = V0
g1
g2
J=

V0 g1 g2
g1 d2 + g2 d1

Reemplazando este u
ltimo valor en los vectores desplazamiento antes construdos y con ello despejando (1):

V0 g1 2
V0 g2 1
+
= l
g1 d2 + g2 d1
g1 d2 + g2 d1

V0
(g2 1 g1 2 ) = l
g1 d2 + g2 d1

b) Esta
nueva situaci
on es de regimen transitorio. Para la interfaz, se tiene:




(
r , t) 6= 0 J2n J1n = l
t
t
La idea es despejar de esta ecuaci
on el termino para la derivada parcial de la densidad de carga libre en la
interfaz. Para esto, se necesitan dos ecuaciones que liguen a los terminos J2n y J1n , y que dicha relacion, (ojal
a),
exhiba terminos en funci
on de l , para luego simplemente resolver la EDO que resulte. Nuevamente, (solo que con
el cuidado de trabajar con dos densidades no necesariamente iguales),

J1
J2
E 1 = k;
E2 =
k
g1
g2
Como el potencial es igual en ambos lados, (pues estan conectados a tierra),
Z
Z

J1 d1
J2 d2

=
( E 1 d
r = E 2 d
r ) (2)
g1
g2
Es importante destacar aqu que el potencial entre la interfaz y las placas conductoras depende del tiempo (pues
est
a conectado a tierra, luego debe descargarse). Por ello no es posible igualar a cero dichos calculos. Por supuesto,
los campos dependen del tiempo, pero no de la posicion. Si hubiera una direccion preferencial de carga/descarga,
habra un campo electrico aplicado distinto de cero en la vecindad de dicha posicion, lo que genera una contradicci
on
con lo recien supuesto de simple conexi
on a tierra.
Nuevamente,

( D 2 D 1) n
= l
J 1 1
J2 2
+
= l (3)
g2
g1
De (2) y (3),

J1

1
2 d1
+
g1
g1 d2


= l

Reemplazando finalmente este u


ltimo valor en cada uno de las densidades de corriente para la interfaz, (ecuaci
on
de continuidad), y utilizando adicionalmente (2),


l = l
t

g1 d2 + g2 d1
1 d2 + 2 d1

l (t) = l (0)et
Con

=

g1 d2 + g2 d1
1 d2 + 2 d1


.

Problema 4
La resoluci
on de este ejemplo plantea inicialmente un sistema de ecuaciones. Se cumplen, por conservaci
on de
energa y de la ecuaci
on de fzas. de Newton,
(1
2
(1)
2 mv = qV
mv 2
= qvB (2)
( x2 )
Donde v corresponde a la rapidez de la partcula, x2 . De aqu, resolviendo,
1 B 2 x2 q
8 V
Tanto el tiempo dentro del sistema como la funcion posicion instantanea son muy sencillas de deducir a partir
de este punto. La u
ltima queda definida despejando de las ecuaciones. Se obtiene que:




 
x
x
8V

i + sen 8V t j
r (t) = =
cos
t
2
2
Bx2
Bx2
m=

Imponiendo (t ) = ,
Bx2
8V
Durante la resoluci
on se supuso, seg
un el enunciado, que la trayectoria es un arco de circunferencia, sin probarlo.
La demostraci
on es muy simple y queda propuesta.
t =

Enunciado Aux. No7 FI2A2


Prof. Aux.: Felipe L. Benavides
Fecha: Lunes 29 de Septiembre de 2008
Problema 1
n Filiforme de B y Ley de Ampe
re
Aproximacio

Para la figura de m
as abajo, cu
anto debe valer r tal que el campo magnetico B en el interior, justo en el centro
del crculo, sea nulo?

Figura No 1
Problema 2
gimen Permanente
Continuidad en Re
La figura de m
as abajo representa un conductor semicilndrico, radio interno a, radio externo b, largo h, con

conductividad g. Una densidad de corriente J fluye entre los contactos rectangulares A y B. (Asumiendolos

propio de
conductores perfectos). Suponga que J , en coordenadas cilndricas, es proporcional al vector unitario ,

estas coordenadas, y cuya magnitud depende de la coordenada , tal que J = j().


a) Determine la dependencia de j() en el radio . (Solo la forma)
b) Obtenga la corriente total I que fluye entre A y B. (Expresela en terminos de lo calculado en a))

c) Obtenga la diferencia de potencial V0 que hay entre A y B y resuma con V0 como dato los valores para J ,

E e I. Calcule adicionalmente la resistencia R del sistema.

Figura No 2
1

Problema 3
re
Aplicaciones Ley de Ampe
Se tiene un cable coaxial ideal de simetra cilndrica que consta de un conductor cilndrico macizo de radio a
rodeado por un conductor cilndrico hueco de radio interior b y radio exterior c = 3a. Por el cilindro central pasa

una densidad de corriente uniforme J 1 = J0 k y por el cilindro hueco exterior circula una densidad de corriente

Determine el campo magnetico en todas partes y calcule que valor debe tener b para que
opuesta, J 2 = 15 J0 k.
este u
ltimo, en la zona exterior, ( > c), sea nulo.

Figura No 3
Problema 4
n de Campos Magne
ticos y Ley de Ampe
re
Superposicio
Se tiene un conductor en la forma de una capa cilndrica recta, infinita, de radio interior a y radio exterior b. Este conductor tiene una densidad de corriente que, expresada en coordenadas cilndricas, es:

J (a b) = + k

Con y constantes conocidas. Obtenga el campo magnetico en todas partes.

Figura No 4

Resoluci
on Aux. No 7 FI2A2
Prof. Auxiliar: Felipe L. Benavides
Fecha: Lunes 29 de Septiembre de 2008
Problema 1
Para resolver el ejemplo calcularemos el campo magnetico producido por cada uno de los componentes del sistema en
el centro del crculo, y luego sumaremos. (Aplicando superposicion). De esta forma, tendremos que para el alambre circular, por definici
on, (usando la aproximaci
on filiforme pues se trata de un cable),
Z

0 I
d
r (
r
r 0)

B( r ) =
0

4 k
r
r k3

Donde, como siempre,


r indica la posici
on en la que se mide, y
r 0 recorre la carga. (En este caso el flujo de
corriente, constante). Parametrizando para el centro del crculo, (colocando all el orgen de preferencia para efectuar
el c
alculo, y teniendo cuidado con el signo del diferencial dada la forma en que se recorre la curva),
0 I

B (0) =
4

Z
0

Rd (0k R
)
3
R

0 I
0 I R2
2 k =
k
3
4 R
2R

Para el alambre rectilneo, usamos por conveniencia, la ley de Ampere, conociendo la direccion del campo previa
mente por el uso de la regla de la mano derecha. (Esta
es general y muy u
til para encontrar la direccion de los campos
en situaciones sencillas). Se tiene que, nuevamente, parametrizando para el centro del crculo, (esta vez colocando el
origen en la proyecci
on r de radio del crculo justo sobre el alambre rectilneo),
Z


B d l = 0 I

B()d = 0 I
0

0 I

B () =
2
Vemos que para calcular la suma de los campos no es necesario adaptar los orgenes, pues, seg
un se tomaron
inicialmente, los vectores unitarios y k coinciden. Evaluando en = r, y planteando el campo magnetico total, se
obtiene:


0 I
1
1

B T (0) =

k
2
r R

r=

Es el valor de r que anula el campo magnetico total en el punto deseado.

Problema 2
a) De la ecuaci
on de continuidad para la corriente en regimen permanente,

J =0
Como

J = gE
Y por lo anterior,

E =0
1 (J)
=0

C

J () =

Con C alguna constante por despejar.


b) Por definici
on, usando (a), (para alguna seccion transversal del sistema),
Z
I=

J dS =

Z
0

Z
a

C
ddz = Chln

 
b
a

c) La diferencia de potencial V0 se obtiene tb. por definicion, como la integral de lnea del campo. As,
Z
C
C
C

g E = J E () =
V0 =
d =
g
g
0 g
Usando V0 como dato y resumiendo, se obtiene:
V0 g
V0
V0 gh

J () =
, E () =
, I =
ln

 
b

V0
=R=
,
a
I
ghln

b
a

Problema 3

Este
ejemplo es una sencilla aplicaci
on de la Ley de Ampere. Para cada trozo, sabemos a priori que el campo
(regla de la mano derecha), y que no depende mas que de . El sistema es infinito en
magnetico apunta seg
un ,
z y simetrico, (dadas las simetras en ), por lo que podemos afirmar que lo anterior se cumple en todos los casos.
Resolviendo,
Z


B d l = 0 I

Donde I es la corriente que cruza la superficie encerrada en la curva , cerrada, que recorre la integral de lnea.
As,
a
Z 2
Z
Z 2 Z
2

B()d = 0
J d S = 0 J0
0 d0 d = 0 J0 2
2
0
0
0
0 J0

B () =

2
ab

Z
B()d = 0

J d S = 0 J0 a2

0 J0 a2

B () =
2
bc
Z 2

Z
B()d = 0





Z
Z

1
1 2

dd = 0 J0 a2 2 b2
J d S = 0 J0 a2

5 0
5
b
h
 2 2 i
0 J0 a2 b
5

B () =

c
Z

Z
B()d = 0





Z
Z

1 2
1 2 c

c b2

dd = 0 J0 a2
J d S = 0 J0 a2
5 0
5
b
h
 2 2 i
0 J0 a2 c b
5

B () =

Para que el campo magnetico se anule en esta u


ltima situacion, (usando que c = 3a)
a2 =

(3a)2 b2
9a2 b2
=
5
5
b = 2a

Problema 4
La u
nica real dificultad que presenta este problema es la forma en abordarlo. Como las corrientes tienen dos
componentes, no puede aplicarse directamente la ley de Ampere, pues es difcil adivinar la direccion que el campo
tomar
a. Por definici
on es difcil tambien, pues el planteo de las integrales es, al menos, complejo. Tiene soluci
on de
esta manera, pero es trabajo innecesario. La idea es que se trabaje para cada componente de la corriente, y con ello
se generen componentes para el campo. La totalidad de ellas, superpuestas, determinan el resultado final. Para cada
caso, por separado, si es posible usar la ley de Ampere, y entonces el problema es muy sencillo. Planteando entonces:
ab


Suponiendo la direccion del campo generado seg
por la regla de la mano
Componente seg
un ,
J = .
un k,
derecha, (y u
nica dependencia en dadas las simetras en z y ),
Z

z0

z0

B()dz = 0
0


0
d
dz
=

ln
z0
0
0
a



B () = 0 ln
k
a


Suponiendo esta vez la direccion del campo generado seg
tb. con la regla de la
Componente seg
un k,
J = k.
un ,
mano derecha, y la misma dependencia espacial para el campo, (por los mismos argumentos anteriores), tendremos:
3


dd = 0

B()d = 0
0

B () = 0

2 a 2
2

2 a2
2


2

As, el campo magnetico total para esta zona es:


  2

 
a2

B () = 0
k
+ ln
2
a
b
El razonamiento es an
alogo, y los c
alculos, muy parecidos. El resultado final es:
  2

  
b
b a2

B () = 0
+ ln
k
2
a
a
Aqu el campo total es cero. Esto no puede solo concluirse de la ley de Ampere, puesto que si una integral de lnea
para una curva cerrada es nula, no necesariamente el integrando es siempre nulo. Eso s, el vector campo magnetico
debe ser nulo si se sabe que no depende de las variables de integracion. Esto, pues si no fuera as la integral no sera
cero. En este caso puede argumentarse que, dadas las simetras, en el mejor caso puede haber dependencia de , pero
nada m
as. Adicionalmente, de la expresi
on general por definicion, dada la ausencia de densidad de corrientes y usando
las simetras del problema, (que permiten separar el desarrollo espacialmente), se concluye similarmente.

Enunciado Aux. No8 FI2A2


Prof. Auxiliar: Felipe L. Benavides
Fecha: Lunes 6 de Octubre de 2008
Problema 1
ticos
Medios Magne
a) Un solenoide de radio 2[cm] y largo 1[m] esta enrollado con un alambre fino a razon de 60 vueltas por
centmetro, transportando una corriente de 4[A]. El interior del solenoide esta lleno de un material paramagnetico
de susceptibilidad magnetica 2.9 104 .

i. Cu
al es la magnitud de H dentro de la substancia?

ii. Cu
al es la magnetizaci
on M dentro de la substancia?

iii. Cu
al es la magnitud de la inducci
on magnetica B , en el interior de la substancia? Cual sera la magnitud

de B si el solenoide estuviera vaco?

iv. Cu
al sera la magnitud de B si en el interior del solenoide se sustituyera el material paramagnetico por
hierro dulce, de permeabilidad magnetica relativa r = 500?
b) Considere
ucleo de hierro dulce, que posee
 otro solenoide, que puede suponerse muy largo, esta vez con n
,
y
que
transporta
una
corriente
de
20
[mA].
Con

e
sta
corriente,
la
permeabilidad
relativa del n
ucleo
2000 vueltas
m
de hierro es 1200. Suponga que se extrae el n
ucleo de hierro. Cuanta corriente es necesario hacer circular por el
solenoide para producir el mismo campo dentro del sistema, que en la situacion inicial?
c) Una esfera de material magnetico de radio R se coloca en el origen de coordenadas. La magnetizaci
on de la
esfera es:


M = ax2 + b i

En que a y b son constantes conocidas.


i. Determine las corrientes de magnetizaci
on.
ii. Ppto: Calcule el potencial magnetico vector y el campo magnetico, en una posicion muy lejana a la esfera
(suponiendo que el radio R no es comparable con la distancia a la que se observa).

Problema 2
Inductancias Mutuas
a) Calcule la inductancia mutua M del circuito de la figura si la bobina 2 posee N espiras, y la 1, n espiras por
unidad de largo.
b) Calcule el coeficiente de acomplamiento k del circuito.

Figura 1
Problema 3
n
Solenoide en Precisio
a) Calcule el campo magnetico sobre el eje de un solenoide de largo L y radio R que es recorrido por una corriente
i, que le da N vueltas.
b) Un cilindro macizo de largo L y radio R esta cargado uniformemente con densidad de carga . Se hace rotar
en torno a su eje con velocidad angular constante w. Calcule el campo magnetico sobre su eje.

Resoluci
on Auxiliar No 8
Prof. Aux.: Felipe L. Benavides
Fecha: Lunes 6 de Octubre de 2008
Problema 1
a)
i. Si se desprecian efector de borde, (i.e., se aproxima el campo dentro de la bobina por el de una muy

larga), el campo magnetico H es, (por Ley de ampere, usando que el campo magnetico fuera de la bobina es
nulo),

H = nI k
Donde k es un vector unitario a lo largo del eje del solenoide, y n es el n
umero de vueltas del enrollado,
por unidad de longitud. La magnitud del campo es finalmente,

 

A
4
k
H = 2.4 10
m

ii. La magnetizaci
on M est
a dada por:

M = m H = m nI k =


6.96

A
m



iii. La magnitud de B es r 0 veces la magnitud de H . Aqu, 0 = 4 107 y r = 1 + m = 1.00029.


Utilizando el resultado de i., queda,

| B |= 3.0168 102 [T ].

Si el solenoide estuviera vaco,

| B |= 0 | H |= 3.0159 102 [T ]

iv. Si se sustituyese seg


un el enunciado,

| B |= 4 107 500 2.4 104 w 15 [T ].

b) Igual que en a), el campo H al interior del solenoide queda

H = N I k
Con I la corriente que circula por el solenoide, y N, el n
umero de vueltas por unidad de longitud del
enrollado. Como en el interior se ha puesto un n
ucleo de hierro dulce, la magnetizacion es

M = m H
(Como la curva de histeresis del hierro dulce es muy delgada, se suele asumir un comportamiento lineal entre

M y H para este tipo de material ferromagnetico, al menos en un tramo suficientemente grande). Finalmente




B = 0 M + H = 0 r H = 0 r N I k = 0 1200 2000 20 103 0.06 [T ] k


Para un solenoide vaco,

B = 0 N I k
Para producir 0.06 [T ], se necesita entonces una corriente de
I=

0.06
23.9 [A]
0 N

c)

i. Este
es un caso muy sencillo. En general se tiene que:
(

J = M (1)

K =M n
(2)
Donde n
es la normal a la superficie. Vemos que la expresion (1) resulta nula, en cambio la (2),



K =M n
= aR2 sen2 ()cos2 () + b sen()k cos()j
Usando coordenadas esfericas, en que
x = Rsen()cos(),
n
= r y r = cosk + sen(cosi + senj).
ii. Basta plantear la integral del potencial magnetico vector en terminos de lo recien calculado y resolver,

suponiendo que el denominador puede salir de ella. De ah, calculando el rotor de A se obtendra el campo

magnetico B .
Problema 2
a) En general, la definici
on de la inductancia mutua plantea que:
M=

12
21
=
I2
I1

donde 12 es el flujo de campo producido por el circuito 2 sobre el 1, y 21 , analogo pero inverso. Dado que
la bobina 2 es m
as ancha y corta que la 1, realizar ley de ampere con la suposicion que el campo solo apunta en
su eje de simetra supone despreciar efectos de borde que son, en principio, no peque
nos. (Ni necesariamente
despreciables). Por lo anterior, trabajaremos con el flujo producido por el solenoide 1, dentro de la 2, i.e., con
el lado derecho de la expresi
on. As,
Z

N
N
M=
B 1 d S 2 = 0 nI1 a2 = N a2 n0
I1 Bobina 2
I1
2

b) El coeficiente de acomplamiento es,


k=

M
L1 L2

De la definici
on para cada autoinductancia,
1
nDB1 a2
=
= 0 a2 n2 D
I1
I1

0 Nd I2 b2 N
N 2 b2
L2 =
=
0
I2
d

L1 =

Finalmente, (despues de un depeje algebraico),


r
a d
k=
b D
Vemos que, si a = b y d = D, i.e., cuando ambas bobinas se convierten en una sola, se cumple que k = 1,
i.e., el acomplamiento es completo, lo que es coherente con el sistema. Si bien se utilizo aqu la ley de Ampere
para obtener el campo dado por cada bobina, debe recalcarse que esto es una aproximacion, en especial para
el caso de la bobina 2, y por lo tanto el coeficiente de acomplamiento aqu obtenido es una aproximaci
on
solamente. Dependiendo de los tama
nos y las proporciones del sistema su validez puede aumentar o disminuir.
Problema 3
a) Para el c
alculo en esta parte no es u
til la ley de Ampere pues la trayectoria cerrada utiliza trozos en
los que no conocemos el valor del campo. Nuevamente la integral por definicion es bastante compleja, y es

preferible trabajar con el vector potencial vectorial y luego tomar rotor. Esto
u
ltimo queda propuesto, puesto
que usaremos en la soluci
on la misma tecnica que en el Problema 3 antes planteado. Calcularemos entonces el
campo magnetico sobre el eje de un anillo circular de radio R, y con el, dividiremos el solenoide en espiras y

sumaremos todos los aportes.


Nota: En rigor, el vector H se denota intensidad de campo magnetico, y el

vector B , s
olo campo magnetico o inducci
on magnetica. En este caso, se calculara H .
Seg
un la figura 1,

Figura 1

i d l
r
dH =
4
r3
Con:

r = z k Rcos()i Rsen()j

d l = (Rseni + Rcosj)d
r 3 = z 2 + R2

 23

Entonces,

i zRcosi + zRsenj + R2 k
d H (z) =
d H (z) =
3
4
(z 2 + R2 ) 2

iR2 k

d H (z) =

2 (z 2 + R2 ) 2

Esto
resulta ser el vector intensidad magnetica sobre el eje z, de un anillo circular de radio R, centrado en
el origen y a altura z = 0. Dividamos ahora el solenoide en espiras de espesor dz 0 . Sea m el n
umero de vueltas
0
0
por unidad de longitud, m = N
.
Entonces,
en
la
espira
de
espesor
dz
habr
a
mdz
vueltas,
y
la corriente total
L
por ella ser
a
i0 = imdz 0
El campo magnetico entonces, (considerando que las espiras estan en z), queda:

d H (z) =

imdz 0 R2 k
3

2 ((z z 0 )2 + R2 ) 2

El campo magnetico total, (producido por todo el solenoide), a la altura z, resulta:

H (z) =

N iR2 k
d H (z) =
2L
z 0 =0

dz 0
3

N iR2 k
=
2L

"

(z z 0 )
p
R2 (z z 0 )2 + R2
#

((z z 0 )2 + R2 ) 2
"

z
zL
N ik

p
H (z) =
2L
z 2 + R2
(z L)2 + R2
0

#L
0

b) Esta
parte es bastante mec
anica, aunque no por eso sencilla. Buscamos nuevamente H , y para obtenerlo planteamos por definici
on y resolvemos. La ley de Ampere tampoco sirve aqu pues nuevamente, dada
la direcci
on del campo, el camino necesario involucra campos de los que se desconoce su valor. Por ello,
parametrizando, tendremos:
Z

J (
r 0 ) (
r
r 0) 0
dV

0
3
k r r k

1
H (z) =
4
Con:



r = z k,
r 0 = r0 + z 0 k = r0 cos()i + r0 sen()j + z 0 k,
J (
r 0 ) = r0 w = r0 w(sen()i + cos()j)
As,

w
H (z) =
4

Z
0

Z
0

(z z 0 )r0 cos()i + (z z 0 )r0 sen()j + r02 k


[(z

w
k
2

z 0 )2

r02 ] 2

ddz 0 r0 dr0

r03 dz 0 dr0
3

((z z 0 )2 + r02 ) 2

Despues de algo de trabajo con esta integral, (primero integrando respecto de dz y luego en dr), se concluye
que:


q

w p 2
2
2
2
2
2
H (z) =
z z + R z + (z L) (z L) (z L) + R
2

Enunciado Auxiliar No9 FI2A2


Prof. Auxiliar: Felipe L. Benavides
Fecha: Miercoles 29 de Octubre de 2008
Problema 1
Se tiene un circuito conductor rectangular, con aristas a(t) = a0 sen(wt) y b, cruzado por un campo magnetico

uniforme y oscilante: B = B0 cos(wt)k que sale de la figura. El circuito esta interrumpido por un condensador de
capacidad C. Desprecie efectos autoinductivos.
a) Determine c
omo vara la carga en la cara 1 del condensador. Suponga que el conductor rectangular tiene
resistencia despreciable. En particular diga cuanto vale la carga en t = 0. Tiene la corriente del circuito la misma
frecuencia w que a(t) y que B(t)?
b) Calcule la carga en el condensador, Q(t), en el caso en que el rectangulo tiene resistencia R, y suponiendo
que la corriente inicial, I(0), es la misma que en el caso anterior.

Figura No 1
Prolema 2
Una varilla conductora, con resistencia R, se puede deslizar por una horquilla de resistencia despreciable, fija en el
espacio, como se indica en la figura. El plano de la horquilla es vertical, y lo atraviesa un campo magnetico perpen

dicular, uniforme, y constante, B . Hay contacto electrico entre la varilla y la horquilla, de modo que constituyen
un circuito electrico cerrado. Si la varilla tiene masa m, calcule la velocidad con que cae, dada la existencia de la
gravedad, si parte desde el reposo. Desprecie efectos de roce, y efectos autoinductivos.

Figura No 2
1

Problema 3
Una espira cuadrada de lado a y masa m puede girar libremente en torno a uno de sus lados, que se elige
como eje z. La espira tiene una resistencia R. En el semiespacio 0 y existe un campo magnetico uniforme,

B = B0i
y en el resto del espacio el campo es nulo. Asuma que el momento de inercia de la espira es In =

5ma2
12

y que se encuentra en el plano y = 0.


a) Suponga que en t=0 la espira tiene velocidad angular
w = w0 k,
Despreciando efectos de autoinducci
on, encuentre la velocidad angular de la espira, una vez que esta ha entrado en
la regi
on y > 0, como funci
on del
angulo entre la espira y el eje i : w(). En particular calcule la diferencia de
velocidad angular que la espira sufre al salir del semiespacio 0 y.
b) Calcule la energa disipada en la resistencia durante el tiempo que la espira permanecio en la regi
on donde
hay campo magnetico.

Figura No 3

Resoluci
on Aux. No9
Prof. Auxiliar: Felipe L. Benavides
Fecha: Miercoles 29 de Octubre de 2008
Problema 1
a) Para obtener la carga en el condensador, en las distintas caras, usamos la conocida relacion C = VQ . Dado que
C es dato, basta encontrar el potencial que enfrenta el condensador, y se tendra lo pedido. Notamos que, dado
que el campo magnetico es variable en el tiempo, y el area del circuito que lo enfrenta tambien, existe inducci
on
de una fem, pues el flujo que atraviesa el
area que encierra el circuito es, en efecto, variable. As, por inducci
on,

=
Por otro lado, el flujo por el circuito es:
I
=

d
dt

B d S = B0 a0 bsen(wt)cos(wt)

Circuito

Como sabemos,
sen(2wt) = 2sen(wt)cos(wt)
Luego,
(t) =

d(t)
d
=
dt
dt

B0 a0 b
sen(2wt)
2


B0 a 0 b
sen(2wt) = B0 a0 bwcos(2wt)
2

La fem que se indujo es negativa. (Siempre es as!). Esto


significa que la corriente que genera dicha fem se

opondr
a al campo magnetico original, B = B0 cos(wt)k. Por esta razon, usando la misma figura, la corriente circuital
ser
a coherente con la regla de la mano derecha para el campo contrario, no para el original. De ah, que la cara
1 reciba una carga positiva, pues el condensador enfrenta potencial positivo desde la cara 1 a la 2. Si rotamos la
fuente, debemos mantener esta convenci
on, pues lo importante no es el dibujo, sino la direccion del flujo de corriente.
Como entra por la cara 1, es positiva para esta cara, y negativa para la 2. Finalmente, la carga inducida en la cara
1 es C V, y en la 2, C V, donde V es la tension inducida en modulo, y no hay ambiguedades, puesto que la
direcci
on del campo magnetico original est
a bien definida.

Figura No 1.
Calculando,
Q1 = +CB0 a0 bwcos(2wt)
Para obtener la carga al instante inicial, basta evaluar en t = 0 la expresion anterior. Se observa que la frecuencia
de la carga, y por ello, de la corriente circuital, es el doble de la variacion del campo. Esto tiene sentido, pues
la variaci
on de flujo est
a directamente relacionada con tanto el cambio en el valor del campo magnetico, como el
cambio en el
area circuital por la alteraci
on de a, en a(t).
b) Planteando la ley de voltajes de kirchoff, se obtiene, (seg
un la figura 2),

Figura No 2.

V (t) = F em(t) = VC + VR =

F em(t) =

Q
+ RI
C

Q
dQ
+R
C
dt

Aqu aparece una edo simple a resolver, pero larga. Se obtendra finalmente que
Q(t) = Q(t)Homogenea + Q(t)P articular
Donde la parte homogenea viene dada por:
t

Q(t)Homogenea = Ae RC
Resolver queda propuesto. La condici
on inicial es la misma que se obtuvo en a), y la solucion particular viene
de proponer una soluci
on de forma similar a la del lado derecho y calcular.
2

Problema 2
Para calcular la velocidad con la que cae la varilla, debemos primero hacer un analisis de fuerzas. Con el, y la
ecuaci
on de Newton, se resuelve todo el problema. La idea es que el campo magnetico genera un flujo variable
sobre el circuito, pues la varilla que lo cierra, se desplaza. La corriente por el circuito, y la interaccion entre ella
y el campo magnetico original, generar
an la fuerza que se opone a su cada. Planteando las fuerzas obtenemos:

F Gravedad = mg k

F M agnetica = I

dl B

V arilla

pues la corriente avanza en


Usando el mismo an
alisis del problema anterior, la direccion del paso dl es seg
un i,
esta direcci
on, dado que el campo que genera, (por la regla de la mano derecha), se opone al original s
olo en esta
disposici
on. De esta forma, integrando sobre la varilla,
Z

F M agnetica = I
dxi (B j) = IlB k.
V arilla

Luego,

F = (mg ILB)k
La corriente eso s, es desconocida. Por ello, usamos la ecuacion que entrega el efecto de induccion,
=

d
d
= ((B)z(t)l)
dt
dt

Como la vara tiene resistencia R,


= RI = Bl

dz(t)
BL d
I=
(z(t))
dt
R dt

d2
l2 B 2 d
(z(t))
=
mg

(z(t))
dt2
R dt

Finalmente, (resolviendo el polinomio caracterstico, (y con el las partes homogenea y particular), sumando y
derivando),
(BL)2
d
mgR
+ Ae mR t
v(t) =
(z(t)) =
2
dt
(BL)
La condici
on inicial es de reposo en t = 0, por lo que,


(BL)2
d
mgR
mR t
(z(t)) = v(t) =
1

e
2
dt
(BL)
Problema 3
a) Necesitamos una ecuaci
on para w(), de tal forma que evaluando obtengamos la diferencia. Para encontrarla, us2
aremos torque, pues de hecho, tenemos el momento de inercia In = 5ma
12 . Como,
X

T = In

Con la aceleraci
on angular, y usando la aproximacion de dipolo magnetico, tendremos,

m B 0 = T = In

Necesitamos entonces
m. Pero
m = I S , donde I es la corriente que atraviesa la espira, y S , el vector de
area
orientada, i.e., el
area orientada seg
un la normal a la superficie. La corriente es desconocida, por lo que usamos
inducci
on. Tendremos que,
Z
((t)) = B0i ddz = B0 a2 sen()

d
((t)) = B0 a2 cos()
dt

I=

B0 a2 cos()
=
R
R

Como S = a2
B 2 a4

k)

In
=
m B 0 = 0 cos2 ()(
R
2 4

B a
In = 0
R

1 + cos(2)
2

Resolviendo la ecuaci
on anterior, y planteando la diferencia entre = 0 y = ,

B02 a4
6 B0 a2
4w =
=
2RIn
5 mR

b) Esta
parte puede resolverse de dos formas, en forma simple. La primera asume el calculo de la potencia disipada
en la resistencia por definici
on, usando adecuadamente las expresiones para la potencia en circuitos sencillos como
este. Otra forma, ciertamente m
as elegante, esta relacionada con darse cuenta que la perdida en energa cinetica
corresponde a la perdida de energa del sistema de la espira en movimiento, y esa perdida puede cuantificarse por
el teorema de trabajo y energa como
4U =

1
1
In wi2 In wf2
2
2

Es interesante notar que la disipaci


on termica de energa por efecto Joule, corresponde, en condiciones ideales,
a la perdida de velocidad angular de la espira. Planteando los valores obtenidos para las velocidades angulares, y
el momento de inercia de la espira, se obtiene el valor pedido.

Enunciado Aux. No10 FI2A2


Prof. Auxiliar: Felipe L. Benavides
Fecha: Lunes 3 de Noviembre de 2008
tica y Fuerzas Asociadas - Electroima
n, Resorte comprimido
Energa Magne
Problema 1
En la figura de m
as abajo se muestra un toroide delgado de seccion circular A, que posee un enrollado de N
vueltas con una corriente I0 . El toroide se compone de dos mitades con permeabilidades magneticas 1 y 2
respectivamente. Dichas mitades se encuentran separadas una peque
na distancia h. (h << a, b). Suponiendo que
el alambre conductor de la bobina tiene una resistencia despreciable, se pide estimar, (lo mas preciso posible),
a) Energa almacenada en el sistema, en regimen permanente.
b) Fuerza sobre la parte derecha del entrehierro, asumiendo que la izquierda esta fija.

Figura 1.
n e inductancias mutuas, el transformador
autoinduccio
Problema 2
Considere un sistema formado por dos bobinas de N1 y N2 vueltas enrolladas en un n
ucleo de fierro toroidal de
permitividad magnetica , seg
un se muestra en la figura. El circuito 1 (de la izquierda) es alimentado por una
fuente sinusoidal, mientras que el circuito 2 se encuentra cortocircuitado.

Figura 2.

Suponiendo que los circuitos 1 y 2 tienen resistencias R1 y R2 respectivamente, se pide:


a) Calcular el valor de las corrientes I1 (t) e I2 (t) cuando ha pasado mucho tiempo desde que se conecto la fuente
de voltaje V (t).
b) Suponga ahora que cuando por el circuito 1 se encuentra circulando la corriente maxima se produce un
cortocircuito, de modo que los puntos A y B quedan unidos entre si en forma instantanea (puede suponerse que
mediante un conductor de resistencia nula). En estas condiciones se pide determinar las corrientes I1 e I2 en funci
on
del tiempo. Que ocurre cuando ha pasado mucho tiempo (t )?

Resoluci
on Aux. No10
Prof. Aux.: Felipe L. Benavides
Fecha: Lunes 3 de Noviembre de 2008
Problema 1
a) Para encontrar la energa almacenada en regimen permanente, encontraremos el valor de la inductancia del
sistema. Basta esto pues, dado que la corriente I0 es un dato, los efectos autoinductivos, si no son despreciables,
ya est
an includos all. Queremos usar,
E=

1 2
LI
2 0

Necesitamos L. Para obtenerla, planteamos por definicion,


L=

I0

Necesitamos el flujo de campo magnetico por el sistema, y para ello, planteamos la ley de ampere. Aproximaremos
que h es suficientemente peque
no como para que el sistema de referencia de la primera mitad del toroide sea v
alido
para la segunda, y cuando desarrollemos, se encontrara que la inductancia depende de la posicion, i.e., no es un

valor fijo. Esto


no permitira que utiliz
aramos la expresion anterior, pero usaremos que el radio del que depende
el valor de L es fijo; el pto. medio del toroide. As,
I


H dl = N I0 = H1 () + H2 ()h + H3 () + H4 ()h
Usando que
B1n = B2n = B3n = B4n = B

B() B()h B() B()h


+
+
+
= N I0
1
0
2
0

B () '

2h
0

N I0

1 +2
a+b
(
)
1 2
2

Asi,
2

L=

N 2 (ab)
4
2h
0

1 +2
21 2 (a

+ b)

Finalmente,
2

E=

1
2

N 2 (ab)

4
2h
0

1 +2
21 2 (a

+ b)

I02

b) La fuerza entre las mitades de toroide es el opuesto a la derivada respecto de la posicion, i.e., h, de la energa
magnetica, recien calculada. Con ello,
2

N 2 I02 (ab)

4
h
i
k=
k F k=k
h
1 +2
0 2h0 + 2
(a
+
b)
1 2
1

Problema 2
a) Para calcular los valores de las corrientes en ambos circuitos en regimen permanente, debemos resolver las
ecuaciones de interacci
on del sistema y de ellas imponer que el tiempo sea muy grande. La idea es usar las
leyes de kirchoff en ambos circuitos considerando el efecto de inductancia mutua que se produce, y la autoinductancia. (La inductancia mutua estar
a dada por la corriente del circuito opuesto, y la autoinductancia, por la
propia. Por ello se plantea el campo magnetico con ambas corrientes). Para plantear dichas leyes, debemos inicialmente obtener el valor del campo magnetico dentro del toroide. Por ley de Ampere, debemos calcular el valor
del campo generado por cada corriente, aproximandolo en todo el toroide por el que hay en el pto. medio de el.
I


H dl = ILibre


H()d = N1 I1 (t) + N2 I2 (t)

a+b
2

(N2 I2 N1 I1 )

H =

(a + b)

El sistema de referencia es cilndricas, con k saliendo de la figura en la hoja. Esto


es arbitrario, y perfectamente
puede suponerse lo contrario y obtener resultados equivalentes. Por otro lado, las direcciones de las corrientes en
cada bobinado tambien son arbitrarias, por lo que puede suponerse cualquier cosa, y, de ser coherente, se llegar
a
a resultados correctos. Usando las direcciones de la figura del enunciado, i.e., con la primera corriente entrando
al sistema, saliendo desde la fuente, y con la corriente en el segundo circuito bajando por el bobinado, la regla de
y el
la mano derecha en cada circuito dice que el flujo de campo generado por el primer circuito apuntara en ,
Planteando las leyes de kirchoff en ambos circuitos, (no asumimos nada respecto de ellas),
segundo, .
V (t) = V0 cos(wt) + Inducida1 = R1 I1
Inducida2 = R2 I2
Las f.e.m. inducidas no son iguales, puesto que el n
umero de vueltas en cada caso no es el mismo. De hecho,
ni siquiera tienen el mismo signo! El c
alculo de los flujos en cada circuito, (inducidos respecto de su opuesto y s
mismos), es donde debemos tener m
as cuidado. Para el primer circuito, la normal apuntara seg
un en nuestro
sistema de referencia. (La regla de la mano derecha para la corriente arbitraria arroja este resultado). Para el
Usamos las direcciones arbitrarias pues el signo menos
segundo circuito, es al reves, y la normal apunta seg
un .
de la inducci
on ajusta despues los resultados. Los flujos finalmente son, (dados los dS vectoriales),
Z
(N2 I2 N1 I1 )


S
1 = N1 B dS 1 = N1
(a + b)
Z
(N2 I2 N1 I1 )


2 = N2 B dS 2 = N2
S
(a + b)
2

donde S = ba
2
La inducci
on queda,
=

Inducida1 = N1

d
dt

N2 I2 N1 I1
S
(a + b)

Inducida2 = N2

N2 I2 N1 I1
S
(a + b)

Podemos comprobar que hemos sido coherentes, puesto que de las expresiones anteriores pueden perfectamente
extraerse los terminos de inductancia mutua y autoinductancia. En cada caso, las inductancias mutuas se suman,
y las propias, se oponen al fen
omeno total de tension en el circuito. (En varios textos se encuentran estas mismas

ecuaciones, pero con el signo cambiado. Esto


se debe a que se asume el signo opuesto dentro de los Inducccion ).
2

Finalmente, las leyes de kirchoff quedan,




V0 cos(wt) + C1 N2 I2 N1 I1 = R1 I1


C2 N2 I2 N1 I1 = R2 I2
N2 S
N1 S
, C2 = (a+b)
usando que C1 = (a+b)
La fsica del problema termina aqu. En adelante, resolver el sistema de ecuaciones permite resolver los comportamientos transientes. Otra forma de deducir estas ecuaciones, eso s, mucho menos trabajosa, es trabajar
directamente con los terminos de autoinductancia e inductancias mutuas.
En general,

Nj jk
ik

Ljk =

donde el subndice j hace referencia al lugar donde se mide el efecto, y el subndice k, a quien lo genera. En
general, los flujos con subndices aluden a los efectos generados por una parte del circuito, cuando la otra no
existe, o no tiene corriente. La superposici
on de los efectos permite resolver en forma mas sencilla. Por ejemplo, la
inductancia L11 se calcula con el flujo que atraviesa el bobinado 1, generado por s mismo, cuando el bobinado 2 no
tiene corriente alguna. La inductancia L22 se calcula con el flujo que atraviesa el bobinado 2, tambien generado por
s mismo, cuando el bobinado 1 no tiene corriente. Finalmente, la inductancia L12 , (equivalente a L21 ), se calcula
con el flujo que atraviesa el bobinado 1, (sin corriente), dado el campo generado por el bobinado 2. Entonces,
L11 =

N12 S
N1 11
=
i1
(a + b)

L22 =

N2 22
N22 S
=
i2
(a + b)

L12 =

N1 N2 S
(a + b)

Usando ahora que,


(V (t)) = [R] (i) + [L]

d
(i)
dt

s
olo basta tener cuidado con los signos. Si se define que ambas corrientes entran al sistema, (ambas bajan por los
bobinados), para ser coherentes con el dibujo y los flujos generados, debemos usar que la autoinduccion es positiva,
y la inducci
on mutua negativa, pues los flujos mutuos son opuestos. As, se obtendra que,
V0 cos(wt) = R1 I1 + L11

0 = R2 I2 + L22

d
d
i1 L12 i2
dt
dt

d
d
i2 L12 i1
dt
dt

que es exactamente lo mismo que obtuvimos al principio. Una forma de resolver este sistema es despejar la
diferencia de las derivadas de las corrientes en la segunda ecuacion, introducir esto en la primera, y derivar dicha

ecuaci
on resultante. Esto
entrega la siguiente EDO,
I2 +

R1 R2
C2 wV0 N1
I2 =
sen(wt)
C2 R1 N2 + C1 R2 N1
C1 R2 N1 + C2 R1 N2
I2 (t) = Aet + Bcos(wt + )
3

Las constantes B y , se despejan de imponer que la solucion es esta, introducirla en la EDO y luego abrir los
cos(a+b) y sen(a+b) con la f
ormula de la suma de angulos. De ah, se tendra que K1 sen(wt) + K2 cos(wt) = 0 con
los K constantes conocidas. Como las funciones sen y cos son ortogonales, las constantes deben ser nulas, para que

la ecuaci
on se cumpla. Esto
despeja y B. La constante A se despeja de la condicion inicial, que es que la corriente

R2
al inicio en el circuito 2 es nula, y es conocida, de la EDO resolviendo la parte homogenea. = C2 R1 NR21+C
.
1 R2 N1
La corriente I1 (t) puede despejarse sin necesidad de construir una EDO para ella. Del sistema, se tiene que,
R1 I1 +

I1 (t) =

C 1 R2 I2
= V0 cos(wt)
C2


V0
C1 R2  t
cos(wt)
Ae
+ Bcos(wt + )
R1
C 2 R1

Al imponer que haya pasado mucho tiempo, las exponenciales caeran, y si la fuente se mantiene, las corrientes
en regimen permanente ser
an sumas de sinusoidales seg
un las ecuaciones para I1 e I2 .
b) La situaci
on queda definida por las siguientes ecuaciones,


+C1 N2 I2 N1 I1 = R1 I1


C2 N2 I2 N1 I1 = R2 I2
De ah,

I1 = I2

I2 +

C 1 R2
C 2 R1

R1 R2
I2 = 0 I2 (t) = De2 t
C2 N2 R1 + C1 N1 R2

R2
. Se ve que la corriente en ambos circuitos se disipara rapidamente en el tiempo.
Con 2 = C2 N2 RR11+C
1 N1 R2
Las condiciones iniciales determinan el problema. La corriente maxima en I1 sale de, despreciando la exponencial,
derivar e igualar a cero en la expresi
on original, considerando la fuente. La condicion inicial determina la constante
D, y de ah el
algebra de las expresiones resuelve el problema en su totalidad.

FI2002 Electromagnetismo
Clase Auxiliar 8
Profesor Auxiliar: Sebastin Fehlandt
Fecha: 26/10/2009

P3 C2 Otoo 2008 Se tiene un sistema formado por dos cables conductores por los cuales
circulan corrientes segn se muestra en la Figura 3.

j
I1

a
I2

Figura 3
Se pide:
a) Demostrar que la fuerza entre alambres paralelos que conducen corrientes de intensidad

I1 e I2, ambas en la misma direccin segn i , es atractiva.


b) Si los dos alambres paralelos son muy largos y estn separados por una distancia a, hallar
la fuerza magntica sobre un segmento diferencial del alambre 2.

FI2002 Electromagnetismo
Clase Auxiliar 10
Profesor Auxiliar: Sebastin Fehlandt
Fecha: 16/11/2009

P1. Se tiene un sistema formado por dos bobinas de


y
vueltas enrolladas en un ncleo de
fierro toroidal de permitividad segn se muestra en la siguiente figura. Los voltajes
y
indicados corresponden a las FEM inducidas en las bobinas, por las corrientes e . Se pide:
a) Suponiendo bobinas ideales, ie sin resistencia. Encuentre una expresin para el voltaje
en funcin de las derivadas temporales de las corrientes e . Haga lo mismo para el
voltaje .
b) Encuentre una relacin entre los voltajes
y
que no depende de las corrientes ni sus
derivadas, slo de los parmetros del circuito.

P2. P1 Examen 2008. Se tiene un par de conductores circulares de rea A y ancho a, entre los
cuales se dispone un medio material con conductividad
,
y
,
segn se muestra en la siguiente figura:

Entre los conductores, usando una fuente de potencial variable, se genera un campo elctrico de
la forma
i.
ii.

. Si

, se pide:

Calcular la frecuencia a la cual el valor mximo del vector densidad de corriente, es


igual al valor mximo de la corriente de desplazamiento.
Cunto vale
?

P3. P2 Examen 2008. Se tiene un cilindro de radio a y altura h (h>>a), el cual tiene una
conductividad g. Se pide calcular lo siguiente:
i.
ii.

Calcular la resistencia del cilindro entres sus dos extremos.


Se desea reducir la resistencia y para ello se rodea el cilindro con una pelcula de cobre
de ancho t. Sabiendo que la conductividad del cobre es gc, se pide determinar la
ecuacin que permite calcular el ancho t de modo de reducir la resistencia en 20%.

P4. P4 Examen 2008. Considere una regin del espacio donde existe un campo magntico
y un campo elctrico
ingresa a este espacio con velocidad
i.
ii.

. Si en t=0, una partcula de masa m y carga q


en la posicin (0,1,0), se pide:

Plantee la ecuacin de movimiento de la carga (en las tres coordenadas)


Determine y dibuje (aproximadamente) la trayectoria que sigue la partcula.

Gua de Ejercicios No 1 FI2A2


Prof. Auxiliar: Felipe L. Benavides
Problema 1
En el plano XY se tiene una densidad de carga que, expresada en coordenadas polares, se calcula seg
un la siguiente
expresi
on: (con a y 0 constantes conocidas)
a3 0

() =

(2 + a2 ) 2
a) Determine el campo electrico sobre el punto del eje Z definido por z = a.
b) El campo sobre el eje Z, con z muy grande, tiende a cero. Determine la forma asintotica, (el orden de magnitud),
con que esto sucede.
Problema 2
Se tiene una esfera de radio R con una densidad de carga uniforme 0 , excepto en una region esferica de radio
b < R, totalmente contenida en la primera, que no posee carga en su interior. Probar que en dicha zona, el campo
electrico es uniforme.
Problema 3
Considere un conductor esferico de radio R con una oquedad en su interior, la que contiene una carga puntual q

seg
un se muestra en la figura. Determine campo y potencial electrico, E (r) y V (r) para r > R. (Ver figura No 1)

Figura No 1
Problema 4

En este espacio se introduce


Considere una regi
on del espacio donde se tiene un campo electrico constante E = E0 k.
una esfera conductora descargada de acero.
a) Dibuje aproximadamente las lneas de campo en las cercanas de la esfera, y al interior de ella.
b) Hay distribuci
on superficial de carga en la esfera?
1

c) Si la respuesta en b) es afirmativa, cu
anto vale el campo producido solamente por la distribucion superficial
de cargas al interior de la esfera?
Problema 5
En la figura se muestra un tubo de rayos cat
odicos como los usados en los televisores. El tubo produce un rayo de

electrones que entran a un espacio limitado entre dos placas. Estas


placas tienen densidades superficiales de carga
dadas por + y , lo que provoca un campo electrico perpendicular a ellas. A una distancia L de las placas se
encuentra una pantalla de largo 2s. Determine tal que los electrones no se escapen fuera de la pantalla. Considere
que el campo electrico es nulo fuera de la regi
on entre las placas, que los electrones ingresan con velocidad v0 en el
eje horizontal y cero en el eje vertical, y que hay gravedad. Asuma conocidas tambien las constantes que aparecen
en la figura, adem
as de la masa y carga de un electron.

Figura No 2
Problema 6

en que k es un vector
Se tiene un campo electrico constante y uniforme en todo el espacio, (vaco), E = E0 k,
unitario seg
un el eje OZ. Se introduce luego una esfera de material dielectrico de constante dieletrica y de radio
R. Como consecuencia de esto, el campo electrico se deforma. Tomese el origen de coordenadas en el centro de la
esfera. LL
amese regi
on I al interior de la esfera, y region II al exterior. Se sabe, (esto es un dato), que la soluci
on
para el potencial en ambas regiones se escribe, en coordenadas esfericas:
(
VI = AI rcos() + BI cos() r12
(r R)
V =
1
VII = AII rcos() + BII cos() r2 (r R)
a) Calcule el valor de las constantes AI , AII , BI y BII . Con ello, obtenga el campo electrico en todo el espacio.
b) Calcule la densidad superficial de carga de polarizacion sobre la superficie de la esfera.

Gua de Ejercicios No 2 FI2A2


Prof. Auxiliar: Felipe L. Benavides
Problema 1
n Temporal de Cargas Libres
Continuidad de la Corriente y Evolucio
Considere un sistema formado por dos placas conductoras conectadas a una diferencia de potencial V0 . En el
espacio interior a las placas se colocan dos medios dielectricos imperfectos. La configuracion, detallada en la figura
1, corresponde a un trozo de circunferencia. Suponga que las placas tienen dimensiones tales que puede despreciar
efectos de borde.

a) Calcule el campo electrico E y la densidad de corriente J entre las placas. Obtenga, asumiendo conocidas
las dimensiones geometricas del sistema, su resistencia total.
b) Calcule la densidad superficial de carga que aparece en la interfaz, en regimen permanente.
c) Suponiendo que la fuente se desconecta del sistema en un instante dado, determine la evolucion temporal de
la densidad de carga libre en la interfaz.

Figura 1.
Problema 2
ticos en Lneas de Transmisio
n
Resistencias y Campos Magne
Parte I
Considere una lnea de transmisi
on compuesta por un cilindro de radio a y altura h, (h >> a), con conductividad
g. Se pide calcular:
a) Resistencia del cilindro entre sus dos extremos.
b) Suponga que se desea reducir la resistencia total, y para ello se rodea el cilindro con una pelcula de cobre, de
ancho t. Usando que la conductividad del cobre es gc , determine el ancho t que permite reducir la resistencia total
del cable en un 20%.

Parte II.
Considere esta vez una lnea de transmisi
on coaxial llena de un material con permeabilidad magnetica no lineal, con
un conductor interno s
olido de radio a y un conductor externo muy delgado, de radio interior b, seg
un se muestra en
la figura 2. Se sabe que en el conductr interno circula una corriente I0 hacia afuera de la hoja, y vuelve en direcci
on
opuesta por el conductor externo. En ambos conductores la corriente se reparte en forma homogenea, y ambos se
1.6H
,
pueden suponer, muy largos. Si la curva de magnetizacion del material se puede aproximar como B = 1000+H
calcule: (El material es el trozo gris de la fig.)
a) Campo magnetico en todo el espacio.
b) Vector magnetizaci
on en el medio material.

Figura 2.
Problema 3
n
Electroima
En la figura de m
as abajo se muestra un toroide delgado de seccion circular A, que posee un enrollado de N vueltas
con una corriente I0 . El toroide se compone de dos mitades 1 y 2 respectivamente. Dichas mitades se encuentran
separadas una peque
na distancia h. (h << a, b). Suponiendo que el alambre conductor de la bobina tiene una
resistencia despreciable, se pide estimar, (lo mas preciso posible),
a) Inductancia del sistema.
b) Energa almacenada en el sistema, en regimen permanente.
c) Fuerza sobre la parte derecha del entrehierro, asumiendo que la izquierda esta fija.

Figura 3.
2

Problema 4
Inductancias Mutuas
Se pretende estudiar la inductancia mutua y algunos fenomenos asociados a ella en la materia, (en forma microsc
opica), modelando el movimiento de los electrones con anillos conductores de corrientes I1 e I2 , para estructuras
moleculares dispares, con uno o m
as
atomos mayores que los otros.
a) Considere dos anillos conductores cuyos planos son paralelos al plano OXY, de un sistema cartesiano de
coordenadas. Los anillos llevan corrientes iguales y en el mismo sentido, y tienen radios a, y b, con a > b. Los
centros de los anillos est
an separados por una distancia d, ambos sobre el eje OZ.

i. Calcule el campo magnetico B en el centro del anillo mas peque


no, producido solo por la corriente que circula
por el anillo m
as grande.
ii. Suponiendo que el campo magnetico es uniforme, (d >> a), en toda la seccion del anillos mas peque
no,
calcule la inductancia mutua entre los circuitos. (Un valor aproximado de ella).
b) Demuestre que la inductancia mutua se puede calcular en este caso, sin aproximaciones, con la siguiente
expresi
on:
M = M12 = M21 =

I I

dl 1 dl 2
r

0
4

C1 C2

donde r es la distancia entre ambos circuitos, (en la integral, dada por la distancia entre la posicion indicada

por dl 1 , para el primer circuito, y dl 2 para el segundo), y C1 y C2 son los circuitos 1 y 2 respectivamente. Es esta
f
ormula general? Si no lo es, en que casos no puede aplicarse?

Figura 4.
Problema 5

lculo de B en diversas situaciones


Ca
a) Calcule el campo magnetico en el origen del sistema de referencia, seg
un la figura de mas abajo. Determine
el valor numerico de lo anterior para I = 2[A] y a = 5[cm].
b) Considere una espira formada por dos semicrculos coplanares concentricos, de radios a y b, con b > a
respectivamente, unidos por dos segmentos como se indica mas abajo. Suponga que por la espira circula una
corriente I. Calcule el campo magnetico producido por la espira en el centro de los semicrculos.

c) Considere un disco aislante de radio a sobre el que se ha depositado una densidad de carga superficial uniforme,
. Si el disco rota en torno a su eje de simetra con velocidad angular w, calcule el campo magnetico producido por
el disco, sobre su eje.

Figura 5.
Problema 6
ctrico y Magne
tico
Cargas en Movimiento en presencia de Campo Ele

y un campo electrico

Considere una regi


on del espaio donde existe un campo magnetico B = B0 k,
E = E0 cos(wt)k.

Si en t = 0 una partcula de masa m y carga q ingresa a este espacio, con velocidad u = u0 i, en la posicion (0,0,1),
a) Determine la ecuaci
on de movimiento de la carga, en las tres coordenadas.
b) Dibuje, en forma aproximada, la trayectoria que sigue la partcula.
Problema 7
Antenas y Lneas de Campo
El alambre indicado en la figura tiene una longitud L y esta recorrido por una corriente I. Calcular, en un punto
P cualquiera situado lejos del alambre (r >> l):

a) El vector inducci
on magnetica, B .
b) A partir del resultado anterior, demuestre que las lneas del campo magnetico en el plano (xy) son circunferencias. Para ello, prosiga como sigue:

i) Demuestre que dr B = 0 corresponde a la ecuacion diferencial vectorial de las lneas de campo magnetico.
ii) Utilice (i) para probar que las lneas de campo magnetico en xy son efectivamente, circunferencias.
4

Para la aproximaci
on, puede basarse en que,
y argumentar en forma razonable.

r =r

12

Figura 6.

z
r

cos() +


z 2
r

Soluciones Gua de Ejercicios No 2 FI2A2


Prof. C
atedra: Sim
on Cassassus
Prof. Auxiliar: Felipe L. Benavides
Problema 1.

1
1
V0
V0

 ;
 ;

a) J () = 
E 1 () =
1 + 2
g1 1 + 2
g1

g2

g1

g2

2
1
g1 + g2

1
V0
R=

 ;
 
E 2 () =
2
g2 1 + 2
(z

z
)
ln
2
1
g1
g2
1


2
V0
1

b) () = 

g2
g1
g11 + g22

c) (t, ) = (0, )et




V0

Con (0, ) =

1
g1

2
g2

2
1

g2
g1


y=

g1 2 + g2 1
1 2 + 2 1

Problema 2.
Parte I.
a) R =

h
ga2


r
g
1
b) t = a
1+
4gc
Parte II.

0 I0

B () = 2a
2
< a

1.6I0
a) a < < b B () = 2000+I0

>b
B () = 0

< a
b) a < < b

> b

M () = 0h

1.6I0
M () = 0 (2000+I

0)

M () = 0

Problema 3.
a) L = h

Problema 4.

I0
2

N 2A
2h
0

(a+b)(1 +2 )
21 2

a) i. B (d) =

0 Ia
3

2 (d2 + a2 ) 2
ii. M =

0 a2 b2
3

2 (d2 + a2 ) 2
b) La f
ormula es general en los casos en que se cumple el teorema de Stokes. As, su validez se acota a las
hip
otesis del teorema, respecto de las propiedades de las curvas y superficies del sistema.
1

Problema 5.

0 I
a) B (0) =
2a

1
1
+
2

Evaluando,

k B (0) k= 2.06 105 [T ]





0 I 1 1
b) B (0) =

4
b a
"
#
1

0 a2 + 2z 2 2 k z k (a2 + z 2 ) 2
c) B (z) =
k
1
2
(a2 + z 2 ) 2
Problema 6.
a) Pendiente.
b)

Figura No 1.
Problema 7.

0 IL
xj yi
a) B '
4 (x2 + y 2 + z 2 ) 23

Gua de Ejercicios No3 FI2A2


Prof. Auxiliar: Felipe L. Benavides
n, generador ideal - torques y movimiento
I. Induccio
Problema 1.1
El generador elemental consiste en una horquilla de separacion d, por la que puede deslizar una barra m
ovil de
masa m y resistencia R. El circuito est
a inmerso en un campo magnetico B perpendicular al plano de la horquilla.
Con el objeto de generar una f.e.m inducida en el circuito cerrado formado por la horquilla y la barra m
ovil, esta
u
ltima se desplaza paralela a s misma, con velocidad constante v.
a) Cu
al es la potencia que se debe entregar a la barra movil para que esta tenga una velocidad constante v ?
b) Cu
anto vale la potencia disipada en la resistencia?
c) Son iguales b) y c)? Que significado tiene esto en terminos de las energas involucradas, y la conversi
on
electromec
anica?

Figura 1.
Problema 1.2
Se tiene una espira rectangular conductora imperfecta de largo l , ancho h, resistencia R e inercia I (en torno al
eje z), que es libre de girar en torno a dicho eje. En cierto instante se somete a un campo magnetico variable en el

tiempo de la forma B = B0 (cos(wt)i + sen(wt)j). Para lo que sigue, desprecie efectos autoinductivos.
a) Encuentre la f.e.m inducida en la espira para todo angulo y para todo instante t debido al campo magnetico
externo. ( corresponde al
angulo entre el plano de la espira y el plano XZ).
b) Encuentre la ecuaci
on de movimiento de la espira.
c) Que sucede con la espira despues de un perodo largo de tiempo? Avale su respuesta con calculos explcitos.

n e inductancias mutuas, el transformador


II. autoinduccio
Problema 2.1
Considere un sistema formado por dos bobinas de N1 y N2 vueltas enrolladas en un n
ucleo de fierro toroidal de
permitividad magnetica , seg
un se muestra en la figura. El circuito 1 (de la izquierda) es alimentado por una
fuente sinusoidal, mientras que el circuito 2 se encuentra cortocircuitado.

Figura 2.
Suponiendo que los circuitos 1 y 2 tienen resistencias R1 y R2 respectivamente, se pide:
a) Calcular el valor de las corrientes I1 (t) e I2 (t) cuando ha pasado mucho tiempo desde que se conecto la fuente
de voltaje V (t).
b) Suponga ahora que cuando por el circuito 1 se encuentra circulando la corriente maxima se produce un
cortocircuito, de modo que los puntos A y B quedan unidos entre si en forma instantanea (puede suponerse que
mediante un conductor de resistencia nula). En estas condiciones se pide determinar las corrientes I1 e I2 en funci
on
del tiempo. Que ocurre cuando ha pasado mucho tiempo (t )?
n temporal
III. Circuitos LRC y evolucio
Problema 3.1
a) Considere un circuito RC serie, (i.e., una resistencia y un condensador conectados en serie), con condici
on inicial
de voltaje V0 . Cu
anto vale el voltaje en el condensador, en funcion del tiempo? Asuma C y R conocidos.
b) Considere un circuito LR serie, (i.e., una resistencia y una inductancia conectadas en serie), con condici
on inicial
de corriente I0 . Cu
anto vale el voltaje en la inductancia, en funcion del tiempo? Asuma L y R conocidos.
c) Considere un circuito LC serie, (i.e., una resistencia y un condensador conectados en serie), con condici
on inicial
de voltaje en el condensador, V0 . Cu
anto vale el voltaje en el condensador, en funcion del tiempo? Y la corriente
por el circuito? Asuma L y C conocidos.
d) Usando las ideas de b) y c), resuelva la siguiente situacion: Se tiene un circuito LC serie, (L1 y C1 ), con
condici
on inicial para el voltaje en el condensador, y con la inductancia acoplada a otro circuito RL serie, (R1 y L1
conocidos), sin circulaci
on de corriente al instante inicial. Cuanto vale el voltaje en la resistencia, si la constante
de acoplamiento entre las bobinas es M ?
tica y Fuerzas Asociadas - Electroima
n, Resorte comprimido
IV. Energa Magne
Problema 4.1
En la figura de m
as abajo se muestra un toroide delgado de seccion circular A, que posee un enrollado de N
vueltas con una corriente I0 . El toroide se compone de dos mitades con permeabilidades magneticas 1 y 2
respectivamente. Dichas mitades se encuentran separadas una peque
na distancia h. (h << a, b). Suponiendo que
el alambre conductor de la bobina tiene una resistencia despreciable, se pide estimar, (lo mas preciso posible),
2

a) Energa almacenada en el sistema, en regimen permanente.


b) Fuerza sobre la parte derecha del entrehierro, asumiendo que la izquierda esta fija.

Figura 3.
Problema 4.2
Un experimento que suele utilizarse para ilustrar la fuerza magnetica es el de la contraccion que sufre un resorte al
hacer circular por el una corriente. Para tal efecto, considere un resorte de constante elastica k y de largo natural l0 .
Supongamos que el resorte est
a hecho de un material conductor, que tiene N vueltas, (espiras), y que es de secci
on
transversal de
area A. Cu
anto se comprime el resorte al hacer circular una corriente I por el?
V. Ecuaciones de Maxwell
Vector de Poynting, Ondas Planas, Intensidad de Ondas, Polarizacion (problemas cortos)
Problema 5.1
Un alambre cilndrico recto de conductividad g y area de seccion transversal A, conduce una corriente uniforme
de intensidad I. Determine la direcci
on y magnitud del vector de Poynting en la superficie del alambre. Integre la
componente normal del vector de Poynting sobre la superficie del alambre, de longitud L, y compare el resultado
con el calor de Joule producido por este segmento.
Problema 5.2
a) Calcule el campo electrico de la radiaci
on electromagnetica en la superficie del sol, sabiendo que la potencia
irradiada por el es de aprox. 3.8 1026 [W ], y que su radio es, tambien aprox., 7 108 [m].
W 
b) En un da soleado, la tierra recibe aproximadamente 1300 m
de energa radiante procedente del sol.
2
Suponiendo que la radiaci
on est
a en forma de onda plana monocromatica, incidiendo normal a la superficie, calcule
la magnitud de los vectores de campo electrico y magnetico.
Problema 5.3
Una onda electromagnetica tiene una frecuencia de 1000 MHz y se propaga en el vaco. El campo magnetico es,
(B0 conocido),

B (z, t) = B0 cos(kz wt)i


a) Encuentre la frecuencia angular, longitud de onda y direccion de propagacion de la onda.

b) Encuentre el campo electrico, E (z, t)


c) Encuentre el vector de Poynting y la intensidad de la onda.
3

Problema 5.4
Una fuente esferica de 40 [W ] emite radiaci
on electromagnetica isotropicamente, de longitud de onda 0 . A un
metro de distancia, esta puede considerarse una onda plana.
a) Escriba el campo electrico de la onda en el punto P de coordenadas x = cos(30)[m], y = sen(30)[m]. Considere
que el campo electrico es perpendicular al plano XY.
b) Calcule la amplitud del campo electrico.
c) Calcule el campo magnetico de la onda, magnitud y direccion.
Problema 5.5
Dos ondas electromagneticas que se propagan en la misma direccion, tienen igual frecuencia, n
umero de onda y
amplitud. Las ondas est
an circularmente polarizadas, en sentido opuesto. Demuestre que la onda resultante est
a
linealmente polarizada, y calcule su amplitud.
Problema 5.6
Describa completamente el estado de polarizacion de las siguientes ondas:

a) E = x
E0 cos(kz wt) yE0 cos(kz wt)

b) E = x
E0 sen(2z/ wt) yE0 sen(2z/ wt)

c) E = x
E0 sen(wt kz) + yE0 sen(wt kz /4)

d) E = x
E0 cos(wt kz) + yE0 cos(wt kz + /2)
Especifique si se trata de ondas planas polarizadas en un plano, elpticamente polarizadas o circularmente polarizadas. Indique la helicidad cuando corresponda.
Problema 5.7
Tres fuentes puntuales de luz monocrom
atica de igual intensidad estan igualmente equiespaciadas sobre una lnea
recta. (Distancia d entre ellas). A gran distancia y paralela a las fuentes se coloca una pantalla. Calcule la
distribuci
on de intensidad de luz sobre la pantalla a lo largo de una recta paralela a las fuentes, en el caso en que
las tres tienen misma frecuencia y fase.
n en medios conductores
VI. Propagacio
Problema 6.1
El campo electrico de una onda electromagnetica que se propaga en un medio conductor, (buen conductor), en la
direcci
on z, tiene la forma:
z
z

E (z, t) = x
E0 e ei( wt)

r
=

2
w

donde es la conductividad del medio y w es la frecuencia angular de la radiacion.

a) Encuentre la diferencia de fase que existe entre los vectores E y B , dentro del medio conductor.
b) Calcule el promedio temporal del vector de Poynting asociado a la onda escrita al principio.
c) Usando a) y b), considere la siguiente situacion: Un equipo de radar sumergido en el mar esta premunido de
un emisor de radiaci
on electromagnetica de 1000 Hz y de un detector que mide la radiacion reflejada por un objeto
 1 
que se interpone en el camino de la onda. Considerando que la conductividad del agua de mar es = 6 m
,
estime la distancia m
axima a que puede estar un objeto para que a
un pueda ser detectado por el equipo. Suponga
que la intensidad de la radiaci
on en el punto de emision es I0 , y que el equipo puede detectar hasta radiaci
on de
intensidad I0 /1000.
Problema 6.2
z
z

Demuestre que la ecuaci


on E (z, t) = x
E0 e ei( wt) satisface ser el campo electrico de una onda electromagnetica
en un medio de buena conductividad. Para ello, escriba la ecuacion de ondas electromagneticas y desprecie el

2
termino E . Puede intentar resolver directamente, o escribir el campo electrico dado y verificar que satisfaga

t2

la expresi
on.
n - Refraccio
n
VII. Reflexio
Problema 7.1
Considere una l
amina de vidrio de espesor d e ndice de refraccion n colocada en el vaco. Sobre ella inciden ondas
electromagneticas en direcci
on normal. Suponga que las ondas incidentes se propagan en la direccion positiva del
eje z y que la l
amina est
a contenida en el plano xy.
a) Escriba la expresi
on del campo electrico y magnetico de las ondas: Al lado izquierdo de la lamina, dentro de
ella y al lado derecho. En estado estacionario, al lado izquierdo y dentro de la lamina existen ondas que viajan en
direcciones +z y z.
b) Escriba las condiciones de borde en z = 0 y z = d, en funcion de los campos especificados en a).
c) Encuentre la amplitud de la onda en la zona d z. Suponga conocida solo la amplitud del campo electrico
incidente.
Nota: Utilice el sistema de coordenadas de la figura de mas abajo, y considere que el vector de campo electrico
de todas las ondas apunta perpendicular al plano de la hoja, y hacia adentro.

Figura 4.

FI2002-3 2009A

Control 1

9 de Abril de 2009

3h

Prof: Simon Casassus


Ayudantes: Mara Jose Santander & Pablo Castellanos

(Desarrolle sus respuestas y cuide la presentaci


on. Sin calculadora. )

I Energa en condensadores.
Consideramos un condensador plano con capacidad C.
1. (1.5pt) Cu
al es la energa U almacenada en el condensador?
2. (1.5pt) Cu
al es la fuerza de atraccion F entre las placas? (ayuda: recuerde que el trabajo en
~ )
un desplazamiento infinitesimal es dW = F~ ds
3. (1.5pt) De una expresi
on para C y F en el caso de placas paralelas infinitas.
4. (1.5pt) En un condensador variable (ver Fig. 1) se puede regular el area entre las placas del
condensador girando una de las placas en una angulo Cual es el torque ejercido en el eje de
un condensador variable? (ayuda: recuerde que el trabajo del torque es dW = d, y aproxime
la capacidad a la del caso plano infinito).

II Cargas superficiales en conductores.


Considere una carga puntual en la vecindad de un conductor conectado a tierra. Calcule la distribucion de cargas en la superficie del conductor, y la fuerza ejercida sobre la carga. Explique las
aproximaciones necesarias. Calcule la carga inducida en el conductor integrando la densidad de cargas
superficial.

III Campos electricos en las superficies de dos esferas conductoras.


El siguiente problema ilustra una manera de entender porque las descargas electricas (i.e. chispas)
se concentran en las puntas de los conductores (i.e. como en los pararayos). Consideramos dos esferas
conductoras de radios a y b, cargas Q y q, respectivamente, y conectadas por un alambre conductor.
Supondremos que las esferas estan suficientementes alejadas para considerarlas aisladas (es decir sin
sufrir la influencias de sus campos electricos mutuos).
1. (1.5pt) Cu
al es el potencial de cada esfera?
2. (2.0pt) De una relaci
on entre Q y q.
3. (2.0pt) Cu
al es el campo electrico en la superficie de cada esfera?
4. (0.5pt) Un i
on, tales como los producidos en las descargas electricas, se encuentra lejos de las
esferas y atrado por ellas. Con que esfera terminara chocando? Justifique.

FI2002-3 2009A

Control 2

7 de Mayo de 2009

3h

Prof: Simon Casassus


Ayudantes: Mara Jose Santander & Pablo Castellanos

(Desarrolle sus respuestas y cuide la presentaci


on. Sin calculadora. )

Relaciones utiles:
~ D
~ = L , D
~ =  E,
~ potencial dipolar: p~ ~r/(4 r3 ) Fuerza de Lorentz: F~ = q(E
~ +~v B).
~

Coordenadas esfericas:
h
i
A
A sin
~ A
~ = 12 r2 Ar + 1
~ = ( , 1 , 1 ),

+
,
r r r sin
r
r
r sin

h
i

 2

2 = r12 r
r2
+
sin

+ 2 .
2
r
r sin

I Esferas dielectricas.
1. (4.0pt) Una esfera dielectrica de radio a, con suceptibilidad E = ( 1) se encuentra
~ = E z.
inmersa en un campo electrico uniforme E
a) Si el campo interno a la esfera es uniforme, cual es el potencial electrico en el
interior?
b) Modelamos la influencia de la esfera sobre su entorno como un dipolo con momento
p = V P , en que V es el volumen de la esfera y P = E 0 Eint es la polarizacion
inducida. Calcule el potencial electrico debido a este dipolo.
c) Exija continuidad en la superficie de la esfera para encontrar el potencial en todo
el espacio.
d ) Confirme por el calculo directo que la solucion anterior satisface la ecuacion de
Laplace, 2 = 0, y confirme que tiene el comportamiento asintotico correcto lejos
de la esfera. Concluya.
e) Confirme que D y Ek son contnuos en la superficie de la esfera.
2. (2.0pt) Consideramos ahora una cavidad esferica de radio a en un dielectrico sometido
~ = E z. Repita el tratamiento anterior para encontrar el potencial electrico en
al campo E
todo el espacio.

II Efecto Hall.
Una intensidad de corriente I pasa por una placa de cobre de seccion rectangular, de largo
~ con direccion perpendicular a la
L, ancho h y altura a, sometida a un campo magnetico B,
placa de cobre (ver figura).
h
a
L

1. (2.0pt) Muestre que la trayectoria de un electron libre en un campo magnetico uniforme,


es un circulo de radio rL = v /L , L = eB/(2m), donde e y m son la carga y la masa del
electron y v es la velocidad inicial del electron. Exprese rL en funcion de I, suponiendo
que la placa tiene un electron por atomo, y que la densidad de atomos de cobre es n.
2. (1.0pt) Explique que sucede con la trayectoria de electrones en los casos lmites L 
rL  h, y rL  h.
3. (2.0pt) Suponga que L  rL  h. Muestre que, en estado estacionario, una medida de
la diferencia de potencial V entre las caras de la placa separadas por h permite inferir B.
4. (1.0pt) Haga una estimacion del voltaje V esperado (sin calculadora), si I = 1 A, y si el
radio de un atomo de cobre es de 1
A, h = 0,5 cm, a = 0,1 cm, e = 1,6 1019 C. Comente.
Como mejorara el dispositivo?

III Aplicaciones de la ecuacion de continuidad de carga electrica.


1. (2.0pt) Deduzca una ecuacion de continuidad que relacione las densidades de carga y
de corriente. (ayuda: Escriba la variacion temporal de carga electrica dQ(t)/dt en un
volumen V como el flujo de la densidad de corriente ~j a traves de S, la superficie que
limita V).
2. (4.0pt) Consideramos una esfera conductora de radio R = 10 cm, inmersa en el oceano
a una profundidad de 10 m. Una corriente I = 1 A escapa uniformemente de la superficie
de la esfera.
a) En estado estacionario, Cual es la densidad de corriente a una distancia r del centro
de la esfera?
b) Si la conductividad del agua salada es = 1 (Ohm m)1 , Cual es la diferencia de
potencial entre la esfera y la superficie?
c) Cual es la resistencia del agua entre la esfera y la superficie?
d ) Cual es la potencia disipada en el oceano?

FI2002-3 2009A

Control 3

11 de Junio de 2009

3h

Prof: Simon Casassus


Ayudantes: Mara Jose Santander & Pablo Castellanos

(Desarrolle sus respuestas y cuide la presentaci


on. Sin calculadora. )

Relaciones u
tiles:
~ = H
~ + M
~
B
~
B = H, = 4 107 T m A1
~ .
~ = B B/
Magnetizacion en materiales lineales: M
R
0
g
~ = ~j(~r0 ) (~r~r0 )3 d3 x0
B
4
k~
r~
rk
~ H
~ = j~l

Energa potencial magnetica de un dipolo:


~
U = m
~ B.
~ + ~v B).
~
Fuerza de Lorentz: F~ = q(E
19
31
e = 1,6 10
C, me = 9,1 10
kg, mp = 1,67 1027 kg.
Rotor en coordenadas cilndricas:
~ A|
~ = Ar Az
~ A|
~ r = 1 Az A

r
z
z
r
~ A|
~ z = A + A 1 Ar

B
h

I Induccion al caer un cuadro por una region con campo magnetico.


~
En la region del espacio contenida entre el plano z = 0 y z = h existe un campo magnetico B
uniforme y horizontal. Inicialmente, en la region z < 0 se encuentra un armazon rectangular conductor
~
de resistencia R, rectangular, indeformable, y de masa m. El plano del armazon es perpendicular a B.
Los lados AB y CD son horizontales y tiene largo a, los lados BC y AD son verticales y tienen largo
b (b < h) (ver Figura). La aceleraci
on de gravedad es ~g .
1. (2.0pt) Establecer, explicando todas las etapas del razonamiento, la ecuacion de movimiento
del armazon.
2. (2.0pt) Resolver la ecuaci
on de movimiento en el caso que el armazon es soltado en t = 0, sin
velocidad inicial, cuando el lado AB coincide on el plano z = 0. Despreciar las contribuciones a
~ debidas a las corrientes inducidas.
B
3. (2.0pt) Cuando el lado DC alcanza z = h, calcular: la velocidad adquirida por el armazon, y la
energa Q disipada en calor dentro del alambre. Comparar Q con la perdida de energa potencial
gravitacional.
4. (+1.0pt) Si existe una corriente I en el armazon, estime el campo magnetico en el centro del
~ infinitesimalmente cerca de uno de lo alambres? Use estos
armazon. Cu
al es la forma de B
~ debidas a las corrientes
resultados para justificar que se pueden despreciar las contribuciones a B
inducidas.

II Usos de un electroiman con sacado.


Consideramos un electroim
an toroidal, con un sacado que subtiende un angulo 1 rad desde el
centro del toro. El toro tiene un n
ucleo de hierro, un radio promedio de a = 10 cm, y el sacado tiene
largo 0,1 cm, y hay 200 vueltas de alambre enrolladas sobre el toro.
1. (4.0pt) Use la siguiente tabla para:
a) estimar el campo magnetico dentro del sacado cuando I = 5 A.
b) Compare la potencia necesaria para mantener un campo magnetico en el sacado de 0.6 T,
con la requerida para mantener un campo de 1.2 T. Comente.
H (A m1 )
B (T)

40
0.1

80
0.2

160
0.6

240
0.85

320
1.0

480
1.2

800
1.4

1600
1.5

2. (2.0pt) Usando el electroim


an estudiamos el compartamiento de materiales no-ferromagneticos.
Ponemos a prueba la relaci
on te
orica
2

m
e2
2
B = n

Zr ,
3kT
6me
en que: n es la densidad de n
umero de dipolos microscopicos, m es el dipolo permanente, r es
el radio promedio de un
atomo, y Z es el n
umero de carga.
a) Cual es la energa magnetica de un grano de material con volumen V , ubicado en el sacado?
b) Comente sobre el comportamiento magnetico de la materia no-ferromagnetica en los lmites
T 0 y T . Deduzca dos tipos de comportamientos magneticos en funcion de los
valores relativos de los par
ametros microscopicos.

III Corrientes de magnetizacion.


Estudiamos las corrientes de magnetizacion en el material aislante que rodea un cable conductor.
Consideramos un cilindro infinito, de radio a, orientado seg
un un eje z, lleno de un material aislante
con permeabilidad magnetica relativa , y con un cable de cobre en su centro. Aproximamos a que la
seccion del cable conductor es nula, es decir asimilamos el alambre a una lnea recta.
~ de intensidad magnetica, en todo el espacio.
1. (1.0pt) De una expresi
on para el campo H,
~ en todas partes.
2. (1.0pt) Escriba el campo magnetico B
~ en todas partes.
3. (1.0pt) Escriba la magnetizaci
on M
~ M
~ , en todas partes salvo
4. (1.0pt) Calcule la densidad de corriente de magnetizacion, j~M =
en la lnea de cobre y en la superficie externa del material.
5. (1.0pt) Use el teorema de Stokes aplicado a un disco
R de radio r < a, y los resultados del Punto 3,
~ en el centro del alambre.
para calcular la corriente de magnetizacion IM = j~M dS
6. (1.0pt) Repita el Punto anterior, pero con r > a, para estimar la corriente neta de magnetizaci
on
en todo el alambre. Compare con la corriente IM llevada en el centro, y concluya.

sP1: Si la densidad total de carga por unidad de largo en la superficie interior es


total en la interfaz es nula entonces el campo para
siempre vale

  
   





 
  


y la densidad de carga


  


!#"%$ y &! "' las que valen


$ * '

 

!"

en la interfaz tiene dos contribuciones,


La densidad de carga de polarizacion

! "%$ )(  $+*  ,  (    ,.- ! "'  * (  /' *  ,  (    ,  


Esto implica que la densidad de carga libre en la interfaz es

 

!10  * $ *  '
   * (  $+*  ' ,    

esferica

sP2: En una configuracion


el campo en cada zona depende tan solo de la carga total encerrada

  ( 43657368 *:9 , <;>A =6  ;@ ? 5 5  2


'

  (CBEDGFHBJICK LMI , 
>
; A =6 N ;@? 5 5 
'

la carga del conductor intermedio. La integral del campo electrico


desde
 ;@? O )esdebe
ser cero y da
9 P
RTS S
S
  ;>A =Q  N;  ?  *
VU =
;WA =6  ; ?
 XO  ;Y?  *Z
;

donde
(con

5  

hasta infinito

* '$ ;

Puesto que caras enfrentadas deben tener cargas opuestas (para que el campo en el interior de los con es
ductores sea cero), la cara interior de la superficie de radio es
y la exterior de ese cascaron
tiene carga
(para que sumen
). A su vez esto significa que el tercer cascaron
en su cara interior
en la cara exterior (para que sumen ).
y carga

'$ ;

* Z ;\[

* ;

'$ ;

Con todo lo anterior en campo electrico


en cada una de las tres zonas no triviales es

5^3 

5
A ;  5 ' -

S
5
*
A ];  5 ' -

S
5

A ;  5 '

El campo para
es nulo y por tanto el potencial es una constante y los potenciales en las tres zonas
que siguen pueden ser escritos, salvo por una constante

;A   52S = 8 $ -

S 8 ;
* A  
5 = '

;A  
S 5

8$ 8'
RTS S
R`S S
S
 ( ^_ 5 _  ,  A ;  5 *  U -  (  _ 5 _ 8 ,  * A ; 
5 *
 U -  ( 8 _ 5 ,  A ; 
5
5   y en 5 
 y que en 5   es nulo. Para 5a3  el potencial, por
Se ve que se cumple continuidad en
continuidad, es la constante
RS S
A ;   * & U

El ultimo

tiene constante nula porque debe anularse en infinito. Exigiendo que el potencial es una funcion
continua se obtiene las constantes y y resulta:

  
$
'

  

sP3: Los campos de desplazamiento son radiales y por tanto normales a la interfaz de radio , lo que
implica que
y que

 
  
$  $  '
'

;
Suponiendo que
la
carga
del
cilindro
de
radio

es
,
la
integral
de
3  3W8 ) se tiene que
de radio  , (
 
 $  9 =

 '  9  ;
que arroja

$=

en una superficie de Gauss cilndrica

'  ; 

Puesto que la componente tangencial del campo electrico


es continua se cumple que

 $$   ''

lo que implica que el campo electrico


tiene una sola expresion

   (  ;  ,  
$= ' 

que implica que la direrencia de potencial es

  (  ;  ,    

$= '
y de aqu la capacidad del condensador es 
 ( $ =  ' , 
 

EJERCICIO 1. FI 2002 ELECTROMAGNETISMO


Escuela de Ingeniera y Ciencias - Universidad de Chile
Prof.
Prof. Auxs.
Fecha:

Luis Vargas
Sebastin Fehlandt
Enrique Guerrero Merino
17 de agosto de 2009

Pregunta
Considere una esfera de carga maciza de radio R y con densidad de carga en
volumen (rv ) = . r, segn se muestra en la Figura 1.
0

Figura 1.
Se pide:
a) Calcule el trabajo necesario para llevar la carga Q desde la posicin inicial
en z=z0 a la posicin final B ubicada en x=a (suponga que a<R) siguiendo
la trayectoria punteada.
b) Calcular la diferencia de potencial entre el punto B y el punto A (VB-VA).
c) Cunto vale el flujo del campo elctrico a travs de una superficie
compuesta de un casquete esfrico de radio 4z0 centrado en el origen?

GAMMBATTE KUDASAI ! (Buena suerte en japons)

You might also like